Сохранен 533
https://2ch.hk/spc/res/227164.html
Домены arhivach.top и arhivach.site временно не функционируют! Используйте домен ARHIVACH.XYZ.
24 декабря Архивач восстановлен после серьёзной аварии. К сожалению, значительная часть сохранённых изображений и видео была потеряна. Подробности случившегося. Мы призываем всех неравнодушных помочь нам с восстановлением утраченного контента!

Тред тупых вопросов #37

 Аноним 10/11/15 Втр 12:35:07 #1 №227164 
14471481075440.jpg
14471481075471.jpg
14471481075482.jpg
Тред вопросов о жизни, Вселенной и всем таком.

Спрашиваем то, за что в других местах выдают путёвку в биореактор. Здесь анонимные ученые мирового уровня критически рассмотрят любые гениальные идеи и нарисованные в Paint схемы.

Прошлый тред https://2ch.hk/spc/res/223277.html
Аноним 10/11/15 Втр 12:36:45 #2 №227167 
>>227164 (OP)
Третий знатно бомбанул от моего комментария на одном научном сайте.
Аноним 10/11/15 Втр 12:56:00 #3 №227174 
14471493601660.jpg
Почему никто не строит ракеты на воде?
Аноним 10/11/15 Втр 12:58:23 #4 №227175 
>>227167
Наш человек.
Аноним 10/11/15 Втр 13:25:09 #5 №227179 
>>227174
Строят. Кислородно-водородные ступени. При горении водорода в кислороде образуется вода. Так что технически такие ракеты летят за счет раскаленного водяного пара.
Аноним 10/11/15 Втр 13:25:45 #6 №227180 
>>227174
Строят, почему. Текущий рекорд высоты аж 800 метров.
Аноним 10/11/15 Втр 13:28:03 #7 №227181 
>>227167
>Третий знатно бомбанул от моего комментария на одном научном сайте.
Ты написал что-то вроде: "Аза-за, затралил лалку!!!11"?
Аноним 10/11/15 Втр 13:28:39 #8 №227182 
>>227174
Если ты про пуск с воды - то гугли "Морской старт". Если про топливо - вода горит плоховато, а кислород-водород загнивающий Запад использует вовсю. Если про постройку на воде - а зачем? Хотя был проект, который планировался к сборке на верфях, но им никто с деньгами всерьез не интересовался вроде.
Аноним 10/11/15 Втр 13:35:38 #9 №227184 
>>227181
Нет, я вообще не пытался никого затралить, просто прокомментировал мимику одного из участников обсуждения, ну а он сразу сагрился(
Аноним 10/11/15 Втр 16:36:06 #10 №227208 
14471625662480.gif
Поясните мне, почему всякие вояджеры запускаются в плоскости эклиптики, почему не запилить перпендикулярный полёт?
Аноним 10/11/15 Втр 16:39:03 #11 №227210 
>>227208
И че там "перпендикулярно" исследовать?
Аноним 10/11/15 Втр 16:43:00 #12 №227211 
>>227210
Облако Оорта хотя бы. Нахуй гнать вояджер через всю эклиптику, чтобы достигнуть края солнечной системы, если с этой же целью можно отправить ракету не вправо, а вверх?
Аноним 10/11/15 Втр 16:43:35 #13 №227212 
>>227208
ну ты уж совсем, как этото вояджер по твоему разгонялся, на термоядерном движке чтоли ? грав маневры с пустотой делать прикажешь ?

Почему с экватора лучше запускать, потому же и в плоскости лучше. заебал ставь ксп.
Аноним 10/11/15 Втр 16:52:02 #14 №227216 
>>227212
>ставь ксп
Ну ты сравнил, это как вместо реальной лиги чемпионов смотреть, как школота в фифу рубится. Ну а хули, команды те же, игроки те же.
Аноним 10/11/15 Втр 16:55:18 #15 №227221 
>>227211
>Облако Оорта
Этого уносите.
Аноним 10/11/15 Втр 16:56:49 #16 №227223 
>>227221
Из треда ТУПЫХ вопросов не уносят, здесь разъясняют.

А кто не может, идёт на хуй сам.
Аноним 10/11/15 Втр 17:00:27 #17 №227225 
14471640277100.jpg
>>227223
Реквестирую эту цитату в ОП-пост следующего треда.
Аноним 10/11/15 Втр 17:01:07 #18 №227226 
>>227211
Потому что гравиманевры. И что ты там в облаке Оорта собрался изучать конкретно? Судя по косвенным данным, если оно и существует, то несмотря на предсказанное обилие объектов, там такая плотность, что посылать аппарат на удачу, наткнуться на что-нибудь и поизучать - это как пытаться выебать ведро.
Аноним 10/11/15 Втр 17:05:27 #19 №227227 
14471643272660.png
>>227226
>Потому что гравиманевры.
Ну вот эту хуйню же как-то отправили к инопланетянам, вряд ли они ожидали, что те их встретят возле Плутона, так что гравиманевры - не панацея. Вот так же отправить перпендикулярно, пусть летит, да информацию скидывает, пока можно. Вдруг эти инопланетяне сверху сычуют, а не за Плуто.
Аноним 10/11/15 Втр 17:07:44 #20 №227229 
14471644646550.jpg
>>227216
Ну так иди играй в реальную лигу. Не можешь? Тогда не выебывайся и ставь ксп, вопросов станет на порядок меньше.
Аноним 10/11/15 Втр 17:07:55 #21 №227230 
>>227223
>Из треда ТУПЫХ вопросов не уносят, здесь разъясняют.
Мод, ты? Как не стыдно врать.

мимоэфирщик
Аноним 10/11/15 Втр 17:08:23 #22 №227231 
>>227223
Тогда иди нахуй.
Аноним 10/11/15 Втр 17:09:29 #23 №227234 
>>227231
Нет ты.
Аноним 10/11/15 Втр 17:10:12 #24 №227235 
>>227227
Господи, да кто ж тебе запрещает, находи бюджет и запускай, как твое душе угодно. Wait o shi~

Ты только что спутал вояджер и новые горизонты или пик тут не причем?
Аноним 10/11/15 Втр 17:10:56 #25 №227236 
>>227227
Да-да, всё финансирование выделили на "вдруг инопланетян встретим?!"

"Вояджер. Новая надежда"
Аноним 10/11/15 Втр 17:12:02 #26 №227239 
>>227230
У вас есть свой просто тупой тред, вот и задавайте там тупые вопросы.
Аноним 10/11/15 Втр 17:16:27 #27 №227241 
Скажите что будет если три или больше тел окажутся рядом в вакууме? Они соединяться или у них будут какие-то траектории вращения?
Аноним 10/11/15 Втр 17:18:38 #28 №227242 
14471651181980.jpg
14471651182001.jpg
>>227208
Потому что орбитальная механика. Если запустить ракету "перпендикулярно", то все равно при выходе на солнечную орбиту траектория полета аппарата будет примерно в плоскости эклиптики. Изменять наклон орбиты это не хуй собачий, очень дорого.
Например, чтобы изучать полюса Солнца, Улисс пришлось пускать на полярную солнечную орбиту с помощью гравитационного маневра у Юпитера.
Аноним 10/11/15 Втр 17:20:36 #29 №227243 
>>227227
Эту хуйню отправили попутной нагрузкой на Пионерах 10 и 11, основной миссией которых было ВНЕЗАПНО изучение Сатурна и Юпитера, а не доставить послание инопланетянам.
>гравиманевры - не панацея
Панацея - не панацея, а позволяют достигнуть более высокой скорости, затрачивая меньше топлива. Новые Горизонты, к слову, запускали "по прямой", с конкретной целью - изучать Плутон. А не "ну хуй знает, может че найдем, а может нет, х.з. короче".
Аноним 10/11/15 Втр 17:40:09 #30 №227254 
>>227241
Они задохнутся.
Аноним 10/11/15 Втр 17:41:36 #31 №227255 
>>227241
Если тела обладают достаточными скоростями, то будут двигаться по замысловатым орбитам, иначе столкнутся друг с другом.
Аноним 10/11/15 Втр 18:09:35 #32 №227256 
>>227164 (OP)
> пик3
Внимательно рассмотри человека на прилагающейся фотографии. Можешь ли ты представить себе, как этот мужчина плачет и набирает дрожащими пальцами телефон какой-то невнятной бляди, чтобы рассказать о своих чувствах к ней? Можешь представить, что он пропускает в форуме жирного тролля с лулзами на уме? А то, что он работает пол-года, раздавая листовки, чтобы купить себе iphone? Ты видишь в нем человека, который стесняется сказать родителям, что на свой двадцать четвертый день рождения он хочет выпить с друзьями?
Теперь посмотри на него еще раз. Видишь ли ты на нем стильные брендовые вещи? Может он покрыт вздувшимися мускулами и толстыми венами? Он обладает внешностью киноактера или мужчины-модели?
Позади него стоит дорогой автомобиль?
Посмотри снова на этого мужчину и спроси самого себя, что с ним не так?
Почему в его умозаключениях железо, в его логике сталь, а вместо эмоций покерфейс?
Аноним 10/11/15 Втр 18:10:24 #33 №227257 
>>227208
Потому что менять наклонение своим ходом, без гравиманевра, крайне невыгодно энергетически. Если только поближе к краю сферы Хилла Солнца, но туда долететь еще надо (там и есть облако Оорта, по сути).

>>227211
Ну направил ты "вверх", как ты до края долетишь? Что так что эдак надо разгоняться. Только лишнюю характеристическую скорость потратишь.
Аноним 10/11/15 Втр 18:12:57 #34 №227258 
>>227256
Попов, залогинься.
Аноним 10/11/15 Втр 18:43:18 #35 №227263 
14471701985230.jpg
>>227258
Нет.
Аноним 10/11/15 Втр 18:45:37 #36 №227264 
>>227263
Астрофизика ответ.
Аноним 10/11/15 Втр 23:17:44 #37 №227347 
Почему при тестировании ракетных двигателей ничего не улетает в космосы и держится стационарно?
Аноним 10/11/15 Втр 23:21:38 #38 №227349 
>>227347
Что не улетает-то, стенд на котором тестируют? А чему там улетать, стенду?
https://www.youtube.com/watch?v=4q48wRsLHVM
Аноним 10/11/15 Втр 23:55:04 #39 №227354 
>>227349
>>227347
Как раз свежий видос подвезли.
http://www.youtube.com/watch?v=0zOzk0keqU8
Аноним 11/11/15 Срд 01:29:00 #40 №227366 
Как выглядит сингулярность и все что с ней связано: бывают ли "открытые" в космосе, сингулярность она одна или их много. Есть ли белые дыры?
Аноним 11/11/15 Срд 01:41:17 #41 №227369 
>>227366
Никто не знает, если ты про биг бэнг, то какбэ никто свечку не держал, если называешь сингулярностью то, что за горизонтом событий ЧД, то тоже невозможно получить информацию, о том что происходит за горизонтом событий.
>"открытые"
Ты о чем вообще?
> Есть ли белые дыры?
Не находили. Хотя два израильских ученых утверждают, что видели взрыв белой дыры. Вроде как сама возможность существования белых дыр - вытекает из решения уравнений, что не всегда так хорошо на практике, как на бумаге Привет струнобогам!.
Аноним 11/11/15 Срд 03:08:42 #42 №227417 
>>227369
>Ты о чем вообще?
О голых сингулярностях, вероятно.
Аноним 11/11/15 Срд 07:35:41 #43 №227432 
>>227417
Да о них. Я и про биг бэнг и про ЧД. Как по мне, эти сингулярности могут вполне быть частью одной условно "гигантской". Но те что в ЧД появляются и которые не видно, они не могут взаимодействовать в противоречии с главной и взрыва не выходит.
Кстати читал про теории конца вселенной там написано что через миллиарды или триллионы лет будет эпоха черных дыр с последующей аннигиляцией всего. То есть по сути возвращение к той самой главной сингулярности, так ведь?
Аноним 11/11/15 Срд 13:20:11 #44 №227462 
>>227432
>так ведь
Так, а может не так. Может все закончится большим разрывом, все это вангоавания на кофейной гуще. Нет понимания темной энергии. Без этого можешь себе любую гипотезу придумать
Аноним 11/11/15 Срд 13:28:24 #45 №227463 
>>227432
>Да о них.
Господствующая гипотеза - что их нет:
https://ru.wikipedia.org/wiki/Принцип_космической_цензуры

А так - никак не выглядит: слишком мала, чтобы ее разглядеть. Только всё вокруг испидарашено.
Аноним 11/11/15 Срд 14:31:17 #46 №227468 
14472414777120.png
14472414777131.gif

Аноним 11/11/15 Срд 14:32:40 #47 №227469 
Блять, ебучие спамлисты, забыл ссылку.
Это>>227468 тебе>>227223
Аноним 11/11/15 Срд 14:40:56 #48 №227470 
>>227468
Устал - отдохни, тебя не заставляют тут дежурить. Устамши он.
Аноним 11/11/15 Срд 15:18:40 #49 №227474 
14472443202850.jpg
>>227470
А кто, если не я? Тут такую хуйню порой несут, аж передергивает. Надо контролировать.
Аноним 11/11/15 Срд 15:42:13 #50 №227477 
>>227474
НУ так контролируй и не ной, думаешь от твоих посыланий нахуй что-то будет, кроме срача и негатива?
Аноним 11/11/15 Срд 15:47:58 #51 №227478 
>>227477
Вполне может быть. Но в следующий раз очередной залетный, желая избежать бурления, пойдет гуглить. Ну если он не упоротый мамкин тралл офк.
Аноним 11/11/15 Срд 15:50:16 #52 №227479 
>>227478
Закроем тред тупых вопросов и заживем сразу. Доску заполнят адекваты и залетные перестанут ходить. Еще матан-капчу у обезьяны вытребуем, что бы наверняка. Так победим.
Аноним 11/11/15 Срд 16:13:16 #53 №227483 
>>227479
Я понимаю, что идеального баланса достигнуть никогда не удастся, но тогда пусть бомбящие бомбят. Нахуя пытаться вразумить кого-то, если тред давно уже мутировал из обычного Q&A в нечто более значимое для доски.
Аноним 11/11/15 Срд 16:19:49 #54 №227485 
>>227478
вот зелень то, какойже залетный откажется от хорошего срача в пользу гугла
сказал бы что школьник решит выебнуться своими знаниями и будет использовать гугл в качестве своей базы знания - я бы охотней поверил.

не будет вопросов - тред будет на дне - такчто задающие вопросы нужны и надо их стимулировать-регулировать. обсирай но отвечай политика. можно ввести коды красный желтый - для анонов зубров. Ой, все, надо спешить, в интернетах опять ктото не прав.
Капитан супер Ответ.
Аноним 11/11/15 Срд 16:55:29 #55 №227499 
>>227485
На дне? Да ебал я такой спейсач, когда в Q&A треде раз за разом спрашивают одно и то же без всякого желания почитать другие треды или хотя бы посты повыше. Уж лучше на дне, чем так, но на плаву. Я все сказал.
Аноним 11/11/15 Срд 17:10:50 #56 №227500 
>>227499
ну я периодически участвую в обсуждениях и краем глаза видел что тут мелькали вопросы по черным дырам, но не принимал в них участие.
У тебя часом не синдром затычки? и ты просто переел плохих вопросов ?

Давай про хабитаты поговорим к примеру? а то чето тихо стало по разделам.
Аноним 11/11/15 Срд 17:27:25 #57 №227503 
>>227174
Еще как вариант игрушечные ракеты - вода-рабочее тело, сжатый воздух источник энергии. См. Водяная ракета
Аноним 12/11/15 Чтв 00:04:49 #58 №227651 
Я ничего не знаю.
Аноним 12/11/15 Чтв 00:07:47 #59 №227652 
Есть ли у фотона масса?
Космос и медицина Аноним 12/11/15 Чтв 00:11:24 #60 №227654 
сап, почаны
учусь в меде, хочу работу связанную с космосом или околокосмосом
какие варианты?
Аноним 12/11/15 Чтв 00:18:18 #61 №227655 
>>227652
Да.
Аноним 12/11/15 Чтв 00:20:54 #62 №227656 
>>227655
Да, на то он и безмассовая частица.
починил себя
Аноним 12/11/15 Чтв 08:17:41 #63 №227744 
>>227654
А ты хочешь проводить медосмотр космонавтов, заведовать тестами для космонавтов или изучать профиты медицины от исследований в невесомости? Пути ведь очень разные, да и спизданул я просто так, ибо не имею представления о системе. Тебе лучше у преподов своих поспрашивать, но для начала спроси себя - готов ли ты к трудному пути по головам и через судьбы ради достижения своей мечты? Или для тебя жизнь обычной потребляди будет предпочтительной? Ответь искренне.
Аноним 12/11/15 Чтв 08:21:31 #64 №227746 
>>227656
Нет массы покоя.
>>227654
Сейчас половина опытов на МКС это медицинские на тему поведения организма в космосе, кости, мышцы, глаза с жидкостями, вот это все.
Аноним 12/11/15 Чтв 08:28:59 #65 №227747 
>>227746
Нет покоя - нет массы покоя. Нет ножек - нет мультиков. Он спросил есть ли масса - есть, и эквивалентности массы и энергии вытекает.
Аноним 12/11/15 Чтв 10:21:41 #66 №227751 
1) При каких условиях будет возможна и будет ли возможна жизнь (а в перспективе и для колонизации человеком) на планете с земного типа, прям вообще, чтобы и g почти такая же и размеры планеты в общем и целом были идентичны. НО при этом планета является не материнской, как в системе Земля-Луна, а дочерней, то есть спутником. Допустим, есть звезда, вокруг которой вращается один газовый гигант, луной которого является наша предполагаемая Земля. Естественно, эта Планета находится в зоне обитаемости. Магнитное поле газового гиганта влияет на эту Планету не больше, чем Солнце на нашу Землю, в следствии разных причин - слабое и не достаточно сильное для выжигающей радиации магнитное поле гиганта или ядро Планеты содержит другой металлосостав, из-за чего собственное поле планеты оказывается в десятки и сотни раз "крепче" аналогичного на Земле, в результат чего атмосфера Планеты по ощущениям человека оказалась бы неотличимой от земной ни по химсоставу, ни по давлению, ни по радиационному фону - ну вы поняли. Это было дано. Теперь вопрос: на каком расстоянии от газового гиганта должна располагать Планета, чтобы не войти спин-орбитальный резонанс 1:1, и чтобы Планета не была направлена в итоге одной стороной к гиганту? А если мы положим Планету боком так, чтобы она смотрела на гиганта не боком, а полюсом? Обеспечит ли это смену сезонов? Будут ли при вращении Планеты вокруг гиганта полюса планеты меняться поочерёдно, периодически сменяясь на посту направления к гиганту или даже если положить полюсом к гиганту, всё равно Планета будет смотреть на гиганта одной только стороной (то есть полюсом)?
2) Исходя из планетарной системы пункта (1) получается, что месяц, с точки зрения наблюдателя на поверхности Планеты, будет определяться не вращение Луны вокруг Земли по небесному своду, а оборотом газового гиганта по небесной тверди Планеты. День будет определяться - как и в случае с Землёй - оборотами Планеты вокруг своей оси. Год, видимо, определяется положением газового гиганта относительно звезды. Так вот, ясен хуй, что газовый гигант - не хуй собачий, а огромная такая елда, так что в зависимости от скорости вращения и орбиты Планеты вокруг гиганта, конкретная точка на Планете может вообще встречать 3 сезона. 1 сезон - лето, когда солнечный свет напрямую постоянно попадает на точку. 2 - зима, когда свет не попадает из-за того, что Планета в данный момент для данной точки отвёрнута от солнца. И наконец 3 сезон - ??????, когда, например, Планета получает энергию не за счёт солнца, а исключительно за счёт теплового излучения с поверхности газового гиганта. Это может быть вызвано тем, например, что газовый гигант, сам по себе очень горячий, ещё и ещё принимает дозу солнечной энергии, в следствии каких-то там внутренних планетарных причин, такой гигант будет настолько горячим, что в определённой точке поверхности, не направленной к звезде, а направленной куда-то вбок (например 160 градусов от ближайшей к звезде точки поверхности) этот горячий газовый гигант будет аж светиться красноватым и излучать крайне мало света, но крайне много тепла. Допустим, получится такой банный сезон, удобный для грибов и растений вообще. Итак, возможна ли такая система с обитаемой дочерней Планетой земного типа с 3 климатическими сезонами вместо 4(2) на Земле?
Аноним 12/11/15 Чтв 10:42:39 #67 №227752 
>>227468
А деваха ничего...
Аноним 12/11/15 Чтв 10:46:33 #68 №227753 
>>227654
>учусь в меде, хочу работу связанную с космосом или околокосмосом
Первый и наиболее важный вопрос: в рашке учишься и живешь?
Аноним 12/11/15 Чтв 10:48:44 #69 №227754 
>>227751
Ебать мои старые костыли...
Аноним 12/11/15 Чтв 11:05:24 #70 №227755 
14473155243600.jpg
>>227754
исчерпывающе
и сразу все ответы всплыли в голове вопросчика
Аноним 12/11/15 Чтв 11:18:32 #71 №227756 
>>227229
Что за порванная дупа на фото?
Аноним 12/11/15 Чтв 11:27:24 #72 №227759 
>>227756
Сегмент бустера SLS после видеорелейтеда.
https://www.youtube.com/watch?v=Qn6OvHofcoo
Аноним 12/11/15 Чтв 11:31:51 #73 №227760 
14473171114770.jpg
>>227752
Да и кот тоже.
Аноним 12/11/15 Чтв 12:27:25 #74 №227764 
14473204459540.jpg
Что будет если во вселенную поместить объект больше всей массы вселенной?
Аноним 12/11/15 Чтв 12:28:55 #75 №227765 
>>227764
Твою мамку шоль?
Аноним 12/11/15 Чтв 12:32:26 #76 №227766 
>>227764
вселенная станет равномерно вращаться вокруг объекта как монолитное кольцо астероидов вокруг сатурна, только это кольцо будет из пространства-времени
Аноним 12/11/15 Чтв 13:17:47 #77 №227769 
Может ли гелий распасться обратно на водород?
Аноним 12/11/15 Чтв 13:41:41 #78 №227771 
>>227769
>Может ли гелий распасться обратно на водород?
Сам нет, но можно разобрать его на кварк-глюонную плазму (как впрочем и любое другое вещество) и из нее соберется водород.
Аноним 12/11/15 Чтв 13:43:53 #79 №227772 
14473250334630.gif
>>227769
Ядро гелия - это одна из самых стабильных частиц. По сути альфа-частицы - это ядра гелия. Что бы они распались надо приложить совершенно неебовое количество энергии. Природный распад такой хуйни не запруфан, насколько я знаю.
Аноним 12/11/15 Чтв 14:04:14 #80 №227776 
>>227771
А можно ли всю энтропию обратить вспять? Может ли через много лет вселенная начать обратно сжиматься и соединить воедино все фотоны (и всё остальное) в изначальное вещество?

>>227772
Спасибо.
Аноним 12/11/15 Чтв 15:28:09 #81 №227823 
>>227751
>возможна и будет ли возможна жизнь
Описывая полную копию Земли, странно спрашивать, возможна ли на ней жизнь.

>на каком расстоянии от газового гиганта
Формула приливного захвата довольно заебная и требует учета многих характеристик. Учитывая, что даже Ганимед, хотя и имеющий довольно невысокую массу, находится в миллионе километров от Юпитера, то можно говорить о сотнях тысячах километрах. Про полюсы пошла какая-то наркомания, извини, даже вникать не стал. Совершенно не зачем боятся приливного захвата. Сейчас объясню:
Что бы находится в обитаемой зоне нужно, что бы газовый гигант был где-то на месте Земли, соответственно, при его массе, орбитальная скорость газового гиганта будет довольно большой. И год газового гиганта будет протекать за несколько недель. Более того, учитывая массу Земли, скорость обращения вокруг газового гиганта будет тоже весьма высокой, так что даже в случае приливного захвата такая планета будет довольно равномерно прогреваться Солнцем. Дни, месяцы, годы будут лететь довольно быстро. Из всего вот этого следует: мало данных. Требуется точно знать массу звезды. Нужно точно знать массу и размер газового гиганта, что бы точно определить его орбитальную скорость и скорость обращения землеподобной планеты вокруг него. Нужно знать наклон орбиты землеподобной планеты, относительно газового гиганта и наклон оси самого газового гиганта. Чем больше данных, тем лучше, тогда можно будет посчитать все по формулам и сказать что-то однозначно, а так - какая-то хуита и гадание на картах таро.
Аноним 12/11/15 Чтв 15:32:10 #82 №227825 
>>227776
Есть гипотеза большого сжатия, есть гипотеза цикличной вселенной, есть гипотеза большого разрыва. Все это гадания, может будет, может нет. Наука не в курсе, мало знаем.
Аноним 12/11/15 Чтв 16:24:50 #83 №227860 
14473346904010.jpg
>>227825
>Все это гадания, может будет, может нет. Наука не в курсе, мало знаем.
Ну, всё же, наиболее популярна сейчас теория, что средняя массовая плотность материи точно равна критической, и, следовательно, пространство топологически плоское и всё завершится тепловой смертью (или Второй инфляционной эпохой, или сработает Теорема Пуанкаре, или случится глобальная больцмановская флуктуация, или реализуется еще один из тысячи сценариев сверх-сверх-далекого будущего).
Аноним 12/11/15 Чтв 16:29:33 #84 №227863 
>>227860
Сейчас одна, вчера другая, завтра залезут в анус темной энергии и перепишут все заново.
Аноним 12/11/15 Чтв 16:32:55 #85 №227865 
>>227863
>темной энергии
Ты эфирщик? Ее природа не так важна. Рассматривай ее просто как коэффициент, приписываемый свойствам пространства.
Аноним 12/11/15 Чтв 16:36:10 #86 №227869 
>>227865
Лолблядь, вы уже определитесь, дауны, у кефироблядей темная материя - костыли, у тебя, ебаната, - эфир.

>Ее природа не так важна.
Чет кипяченые с тобой не согласны ни хуя. Им почему-то важна.
Аноним 12/11/15 Чтв 17:12:45 #87 №227880 
>>227869
Не т. материя, а т. энергия. Сейчас принято считать, что это просто константа. Ее природа важна, несмотря на авторитетное мнение выше, поскольку позволит предсказать, будет ли изменение скорости расширения постоянным и, следовательно, какой из сценариев смерти вселенной наиболее вероятен.
Аноним 12/11/15 Чтв 17:20:23 #88 №227884 
>>227869
Давайте только без оскорблений.
другой анон
>>227880
Мне кажется, что темная энергия как волна, вначале расширение ускоряется, потом замедляется, а потом идет обратный процесс.
Аноним 12/11/15 Чтв 17:24:09 #89 №227889 
>>227880
>это просто константа
>константа
>позволит предсказать, будет ли изменение скорости расширения постоянным
>постоянным
Аноним 12/11/15 Чтв 17:40:49 #90 №227892 
>>227860
Это фотожоп?
Аноним 12/11/15 Чтв 17:43:30 #91 №227893 
>>227892
Любая астрофотка проходит обработку в фотошопе. Так что вопрос некорректен.
Аноним 12/11/15 Чтв 17:53:57 #92 №227894 
>>227893
Хорошо, это слишком похоже на снимок телескопа.
Аноним 12/11/15 Чтв 17:55:29 #93 №227895 
>>227894
Снимки с телескопа тоже проходят обработку в графических редакторах.
Аноним 12/11/15 Чтв 17:57:54 #94 №227896 
>>227893
Не любая. Такое фото можно сделать выдержкой+фишай.
Аноним 12/11/15 Чтв 17:58:40 #95 №227897 
>>227895
Блядь, так это настоящее фото или снимок телескопа склеенный с панорамой каньона?
Аноним 12/11/15 Чтв 17:59:45 #96 №227898 
>>227897
Это настоящее фото с крутящегося вслед за звездами фотоаппарата и выдержкой в очень много секунд.
Аноним 12/11/15 Чтв 18:01:06 #97 №227899 
>>227896
Можно, а потом ее все равно обрабатывают в фотожопе.
Аноним 12/11/15 Чтв 19:09:52 #98 №227914 
>>227889
Ну и что ты тут пытаешься сказать? Хорошо, если хочешь как с закадровым смехом - держи:
Сейчас принято считать, что это просто константа. Но существует как минимум одна альтернативная версия - что это своего рода частицеподобные возбуждения некоего динамического скалярного поля, называемого «квинтэссенцией». Отличие от космологической константы в том, что плотность квинтэссенции может варьироваться в пространстве и времени Ее природа важна, потому что сейчас мы имеем предположение, которое с высокой степенью вероятности может быть верным. Достаточно разжевал? Или опять будешь мои же строчки копипастить?
Аноним 12/11/15 Чтв 19:15:01 #99 №227918 
>>227914
Скорость расширения пространства непостоянна.
Аноним 12/11/15 Чтв 19:30:38 #100 №227930 
У меня в голове до сих пор не может уложиться - самые далёкие галактики что мы видим находятся на расстоянии 14 млрд. световых лет от нас, т.е. они такими были по состоянию 14 млрд. лет назад, а какие они сейчас?
Аноним 12/11/15 Чтв 19:39:20 #101 №227934 
>>227918
>изменение скорости расширения
>изменение скорости
>изменение
Аноним 12/11/15 Чтв 19:42:22 #102 №227935 
>>227918
>т. энергия. Сейчас принято считать, что это просто константа.
>темная энергия константа
>темная энергия
Заодно.
Аноним 12/11/15 Чтв 19:44:11 #103 №227936 
>>227930
Самые далёкие в данный момент находятся гораздо дальше, 46, вроде бы.
Аноним 12/11/15 Чтв 20:24:39 #104 №227942 
>>227930
>какие они сейчас
Сейчас это 12 ноября 2015 года? Через
>14 млрд.
лет узнаем.
Аноним 12/11/15 Чтв 20:37:18 #105 №227945 DELETED
>>227942
Вот для таких долбоебов, как ты, и ясно постулировали неотделимость пространства от времени. Нет никаких "как они там, бедненькие?". Всяким пидорасам трудновато понять, что такое "информация" и "пространство-время", а посему - да начнется срач.
Аноним 12/11/15 Чтв 20:47:01 #106 №227947 
14473504211010.jpg
>>227945
Аноним 12/11/15 Чтв 20:50:12 #107 №227949 
Обьясните теорию сингулярности, ибо вики херь какую-то выдет
Аноним 12/11/15 Чтв 20:52:39 #108 №227951 
>>227945
"Рост мудрости можно точно измерять степенью уменьшение злобы."
Аноним 12/11/15 Чтв 21:00:22 #109 №227956 
>>227951
>можно точно измерять степенью уменьшение злобы
Моя есть хороший китайца. Моя хорошо говорить русский.
Аноним 12/11/15 Чтв 21:04:28 #110 №227963 DELETED
>>227956
Sic!
Аноним 12/11/15 Чтв 21:09:52 #111 №227966 DELETED
>>227963
Моя хорошо знать два-три [забыл слово] латинянских выражения!
Аноним 12/11/15 Чтв 21:17:35 #112 №227969 
зачем сублимировать еду если в космосе один хрен нет воды и приходиться её тащить туда отдельно?
Аноним 12/11/15 Чтв 22:23:22 #113 №227992 
>>227969
В данном случае сублимация = консервирование.

проклинаю новую капчу-кун
Аноним 12/11/15 Чтв 22:53:18 #114 №228011 DELETED
Чувствую себя самым одиноким во вселенной.
Аноним 12/11/15 Чтв 23:17:44 #115 №228026 DELETED
>>228011
Чувствую, политическая "Партия одиноких людей" выйгровала бы все выборы, во всех станах, всегда.
Аноним 13/11/15 Птн 00:25:01 #116 №228049 DELETED
>>228026
Если столько одиноких людей, почему они вместе не сгруппируются и не станут одинокими?
Аноним 13/11/15 Птн 00:45:27 #117 №228056 DELETED
>>228049
Они безынициативные слизни сидящие дома.
Аноним 13/11/15 Птн 01:40:22 #118 №228059 
>>227164 (OP)
Почему так много углеродных ханжей?
Аноним 13/11/15 Птн 01:43:20 #119 №228060 
>>228059
Ты про углеродный шовинизм? Потому что знаем только такой тип жизни, а для него углерод лучший кандидат, остальные уступают ему. Хотя жизнь может строиться на незнакомых нам принципах, но тогда невозможно строить предположения где она может быть и какой.
Аноним 13/11/15 Птн 03:38:12 #120 №228068 DELETED
>>228056
Тебе с нами не по пути, грязный социоблядок
Аноним 13/11/15 Птн 04:15:18 #121 №228070 DELETED
>>228068
Тебе вообще ни с кем не по пути, амёба.
Аноним 13/11/15 Птн 07:20:08 #122 №228076 
>>228060
Почему другие уступают? Разве нет в периодической таблице другого похожего по возможностям элемента, как углерод? И вообще, формы жизни из тяжелых металлов были бы более надежными. Просто им сложней создаться в естественных условиях. Но человек бы мог.
sageАноним 13/11/15 Птн 08:58:36 #123 №228090 
>>228076
>Разве нет в периодической таблице другого похожего по возможностям элемента, как углерод?
Нет, такого, как углерод нет. Только он способен давать такое разнообразие молекул, особенно полимерных. И вообще нахуя природе базироваться на чем-то другом, когда есть божественный углерод, которого пиздос как дохуя и который имеет такие возможности?

>Cложные, нерегулярные и в то же время высокостабильные структуры получаются только с помощью элементов, способных связываться между собой прочными ковалентными связями в длинные ветвящиеся цепочки. На эту роль подходят только неметаллы с валентностью больше двух (для ветвления цепей): бор, углерод, азот, кремний и фосфор. Но бора при термоядерном синтезе в естественных условиях образуется очень мало, азот способен к полимеризации только при сверхвысоких давлениях глубоко в недрах планет-гигантов, до которых вряд ли когда-либо удастся добраться, а элементы третьего периода — кремний и фосфор — формируют стабильные цепи только при чередовании с кислородом, причём гораздо более массивные и склонные к образованию однообразных сетчатых (двухмерных) полимеров. Кроме того, эти элементы в цепи не обладают способностью образовывать кратные пи-связи, совершенно необходимые для сборки плоских структур, поэтому «чисто-силоксановая» жизнь невозможна — совсем без углерода обойтись не удастся и максимум возможного здесь — примесь кремния в гетероорганических молекулах: органосилоксанах и, возможно, органосилатранах.
А фосфор и так уже запользован нашей жизнью для связи нуклеотидов в нуклеиновых кислотах и одновременно как оперативное хранилище химической энергии в виде свободных нуклеотдов АТФ и ГТФ.

Вот ты идешь по пустыне и хочешь жрат и пит, видишь оазис, там вода чистая, жрачка растет на деревьях, прямо вот руку протяни, рви и жри, и так далее, а ты такой: "ну нахуй, это все как-то просто, я лучше в пустыне ящериц наловлю и из них поем и попью."
Аноним 13/11/15 Птн 09:04:45 #124 №228094 
>>228077
DEUS EX /b/, ti veni, vidi, huesosi suda prishel, suka?
>>228076
Только собрался писать, как этот мудрый господин >>228090 опередил. Есть еще кремний и кислород, но кремнию толще и тяжелее, ему труднее в связи (половые и двойные и тройные ковалентные), азот и фосфор (тоже со связями сложнее), азот и бор... И соответственно растворители могут быть не водой (аммиак, фтороводород).
Аноним 13/11/15 Птн 10:01:54 #125 №228103 
>>227823
> Описывая полную копию Земли, странно спрашивать, возможна ли на ней жизнь.
Ну мало ли.
> Из всего вот этого следует: мало данных. Требуется точно знать массу звезды. Нужно точно знать массу и размер газового гиганта, что бы точно определить его орбитальную скорость и скорость обращения землеподобной планеты вокруг него. Нужно знать наклон орбиты землеподобной планеты, относительно газового гиганта и наклон оси самого газового гиганта. Чем больше данных, тем лучше, тогда можно будет посчитать все по формулам и сказать что-то однозначно, а так - какая-то хуита и гадание на картах таро.
Допустим, звезда размера Бетельгейзе. А размер газового гиганта в 3-10 раз больше Юпитера. Наклон орбиты землеподобной планеты, допустим, от 20 до 45, для обеспечения смены времён года по земному типу, а наколон гиганта я не могу сказать, потому что не представляю, особо-то, как его наклон может повлиять на Планету. Ясное дело, что полюсарной шершавой радиацией по щекастой поверхности Планеты он не должен водить, так что не более 70, я думаю. Наколько Планета должна быть близко? Вообще представляю себе картину, при которой четверть неба могла бы быть спокойно перекрыта телом ГГ, с красноватым тёплым свечением, но никак не продумать, будет ли при таком положении Планеты при ГГ возможность существования указанного трисезонья
Аноним 13/11/15 Птн 12:05:37 #126 №228150 
>>228103
>как его наклон может повлиять на Планету.
Хм, ну например так, что все крупные спутники вращаются в плоскости экватора планеты-хозяина?
Аноним 13/11/15 Птн 13:12:30 #127 №228202 
14474095500920.jpg
>>228150
Не совсем, но близко к тому.
Аноним 13/11/15 Птн 16:05:00 #128 №228282 
14474199006390.jpg
>>227474
> кто, если не я?
Ты прям как тот цыган из анекдота.

> Надо контролировать.
Думаешь тебе это зачтется где-нибудь?

> хуйню порой несут
Ничего не случится если кому-то придется побампать свой воброс. А интернет все равно останется огромной информационной помойкой, сколько её ни чисти.
Аноним 14/11/15 Суб 00:19:39 #129 №228499 
>>228202
>>228150
И как оно может повлиять? Для Планеты-спутника Газовый Гигант всё равно всё время будет по экватору
Аноним 14/11/15 Суб 10:09:54 #130 №228547 
14474849944910.jpg
>>228282
Но когда-то этот тред был не столько помойкой спейсача, сколько его образовательным центром. А "тупые вопросы" задавали люди с нехилым таким базисом физики. Да я и сам в этом треде дополнил свою картину мира и восполнил множество пробелов, оставшихся после прохождения университетского курса. А сейчас уровень дискурса в треде сравним с оным в комментариях к ролику на тытубе с названием "черная дыра - убийца миров!".
Это печально.
Аноним 14/11/15 Суб 10:45:44 #131 №228548 
>>228547
Ты ще вспомни, что когда-то двачи были не пабликом вк.
Аноним 14/11/15 Суб 10:52:44 #132 №228550 
14474875646750.jpg
>>228548
Ты ещё вспомни, когда двач был тот.
Аноним 14/11/15 Суб 10:54:50 #133 №228551 
>>228547
>А "тупые вопросы" задавали люди с нехилым таким базисом физики.
Да не сочиняй, всегда были тупые вопросы один тупее тупого и вопросы с базой, ничего не поменялось и сейчас точно так же. Может за исключением первых нескольких тредов, когда мы тут между собой друг у друга спрашивали.
Аноним 14/11/15 Суб 15:06:29 #134 №228596 
Пока обедал, назрел тупой вопрос: как, хотя бы теоретически, можно нейтронную звезду разрушить или иначе прекратить сжатие, чтобы посмотреть, вернется ли вещество из вырожденного нейтронного газа в обычное? Все варианты которые пришли в голову - херня. Радиус слишком мал, чтобы мимокрокодилящий компактный объект разорвал приливными силами (или надо в упор ЧД запускать на большой скорости), закидывать веществом бесполезно, в завсисмости от типа н.з. она или сблеванет его джетами, или просто возьмет и не вернет (как друг в школе взял мой диск с фалаутом 2 от 7 волк), пытаться выстрелить на релятивистких скоростях 20-30 км железными ядрами? Это уже мой мозг выдал, когда не осталось нормальных версий.
Аноним 14/11/15 Суб 16:30:05 #135 №228627 
>>228596
Слишком быстрое вращение?
Аноним 14/11/15 Суб 16:45:15 #136 №228633 
14475087159440.jpg
Что если создать подземную базу на Венере? Как глубоко она должна находиться?

Или же там еще больше давление получится и венеротрясения все разрушат?
Аноним 14/11/15 Суб 16:50:41 #137 №228635 
>>228633
Прежде чем пилить где-либо что-либо, необходимо изучит местность, а что бы запилить что-либо под поверхностью даже на Земле проводят 30 экспертиз и дрочат геологию. Полетели, хуле запилим на Венере что нам стоит. Хули там делать на этой Венере?
Аноним 14/11/15 Суб 17:03:56 #138 №228638 
>>228596
ну для возврата вещества из этого состояния достаточно снизить плотность, это верно. Только вот отобрать у нейтронки (электронки, белого карлика - у кого угодно из них) её законную материю - почти нереально. А вот "незаконную" - ты прав, сама сблюёт. Только теперь внимание вопрос - а какая разница? Ведь "незаконное" вещество перед выбросом джета дойдёт до нужной плотности, а значит - мы будем наблюдать трансформацию вырожденного нейтронного-электронного газа (зависит от звезды) в нормальную плазму. Правда, плазма будет нейтронной, а не железной, и распадется потом до протонов и электронов - то есть водорода.
Аноним 14/11/15 Суб 17:39:09 #139 №228648 
>>228635
Разве вопрос был в том, что нам делать на Венере? Зачем ты со своим прагматизмом и ответными вопросами лезешь?
Аноним 14/11/15 Суб 18:37:45 #140 №228672 
>>228638
А точно дойдет? Оно же "доходит" только под 1-2 км поверхности самой нейтронной, а джеты разгоняет магнитными полями, так что там увеличивается скорость, что далеко не всегда дает увеличение плотности и вырождение в нейтронный газ.
Аноним 14/11/15 Суб 18:41:26 #141 №228675 
>>228648
Съеби в сайфай и зарывайся хоть на Меркурии, аутист.
Аноним 14/11/15 Суб 19:34:32 #142 №228684 
Я читал про теорию инфляции и репульсивную гравитацию и наткнулся на вот это предложение.
>Современная теория элементарных частиц достаточно убедительно указывает на то, что при очень высоких энергиях могут существовать такие формы материи, которые создают отталкивающую гравитацию.

Как выглядят эти самые формы материи? Из каких частиц они состоят? При каких энергия образуются?
Аноним 14/11/15 Суб 19:58:27 #143 №228687 
Почему одни атомы относительно стабильны, а другие быстро распадаются? Почему атомам спокойно не сидится?
Аноним 14/11/15 Суб 20:17:04 #144 №228689 
Насколько правдивы эти предположения?
https://ru.wikipedia.org/wiki/%D0%92%D1%80%D0%B5%D0%BC%D0%B5%D0%BD%D0%BD%D0%B0%D1%8F_%D1%88%D0%BA%D0%B0%D0%BB%D0%B0_%D0%B4%D0%B0%D0%BB%D1%91%D0%BA%D0%BE%D0%B3%D0%BE_%D0%B1%D1%83%D0%B4%D1%83%D1%89%D0%B5%D0%B3%D0%BE
Аноним 14/11/15 Суб 20:21:55 #145 №228690 
>>228687
Потому, что энергия образовавшихся элементов меньше, чем энергия исходного атома.
Если протонов много-будет альфа распад (вылет ядра гелия),
Если много нейтронов - бетта минус распад - вылет электрона.
Ну в добавок ещё и гамма излучение.
Остальные варианты встречаются реже.
Аноним 14/11/15 Суб 20:25:24 #146 №228691 
>>228689
С некоторой вероятностью. Чем дальше прогнозируешь, тем размытей будущее.
Аноним 14/11/15 Суб 20:46:21 #147 №228693 
>>228690
А наименьшая энергия это какая? Когда атом становится водородом? Вот например гелий-3 и гелий-4, оба стабильны. Как так, неужели они оба находятся в состоянии с наименьшей энергией? Или они всё-таки условно стабильны, а их нестабильность настолько мало, что ей пренебрегают?
Мне казалось что распад связан с вероятностью тунеллироваия барионов через кулоновский барьер.
Аноним 14/11/15 Суб 20:48:51 #148 №228694 
>>228672
Ну если взять за основу твоё предложение с релятивистскими снарядами - то дойдет. Всё ж нейтронка, не черная дыра... Другое дело, что новые нейтроны не приживутся в звезде (там же вырожденное вещество, т.е. все энергоуровни заполнены родными нейтронами). Ну и отправятся в полет джетом. Большая часть джетов и образована теоретически тем веществом, что таки добралось до звезды, но ему сказали "Звезда нерезиновая" и швырнули назад))) Ну а там уже магнитные поля сформировали "поезд эмигрантов" и на мороз)
Аноним 14/11/15 Суб 21:03:25 #149 №228699 
>>228693
Где то в районе железа. Есть диаграммы дефекта массы атома (на сколько он легче суммы масс составных протонов и нейтронов), там максимум где-то в этом районе.
Есть диаграмма где указаны стабильные атомы.
Аноним 14/11/15 Суб 22:19:29 #150 №228732 
>>227164 (OP)
Перед моим походом в реактор скажите: как отличить растущую луну от умирающей?
Аноним 14/11/15 Суб 22:25:44 #151 №228743 
>>228732
Если она похожа на "С" - стареющая, если на полукруг от "Р" - растущая.
Аноним 14/11/15 Суб 22:45:49 #152 №228752 
>>228732
Учитывая упоминание реактора - умирающая Луна разваливается на куски и часть из них уже чертит красивые огненные линии в атмосфере.
Аноним 14/11/15 Суб 22:50:03 #153 №228755 
14475306032900.png
14475306032911.png
14475306032912.jpg
>>228684
Кто-нибудь? Я так понял, что у такого материала отрицательная плотность. Это как-то связано с натяжением вакуума?

И еще. Согласно этой теории(инфляции) пространство расширяется при постоянной плотности, образовывая энергию в форме материи. А за счет чего она появляется? За счет флуктуаций поля? Тогда какого?

Ответьте плес.

как вообще уложить в голове понятие "отрицательная плотность", это первое понятие, которое я вообще не могу себе хоть как-то представить
Аноним 14/11/15 Суб 23:09:47 #154 №228768 
>>228755
>отрицательная плотность
Это значит, что тело способно плавать на поверхности вакуума.
Аноним 14/11/15 Суб 23:12:16 #155 №228772 
>>228755
Ну, просто квантовая теория допускает симметрию по нескольким параметрам - по заряду (материя-антиматерия), по массе (как раз про античастицы из этой симметрии в том посте и речь), по временному показателю вроде тоже... Прикол в том, что частицы с отрицательной массой тоже не могут достигнуть скорости света, но... с другой стороны. То есть скорость света для них - наименее возможная.
Аноним 14/11/15 Суб 23:39:20 #156 №228789 
>>228694
>Большая часть джетов и образована теоретически тем веществом, что таки добралось до звезды, но ему сказали "Звезда нерезиновая" и швырнули назад
Дык нет же, джеты сформированы заряженными частицами (невырожденными), которые "эжектирует" сама звезда, в магнитном поле они превращаются в плазму, разгоняются до околосветовых скоростей (благодаря тому, что магнитное поле вращается вместе с нейтронной звездой и на определенном радиусе достигает скорости света), и выбрасываются с полюсов. Если она будет вращаться недостаточно быстро для эжекции и свечения в рентгене, но не достаточно медленно, чтобы аккрецировать, вещество - все еще не сможет достигнуть ее из-за магнитного поля, но и не будет наблюдаемых проявлений. Все еще не вижу, как это вещество ужмется в нейтронный газ.
Зато родил еще одну интересную идею, прокатит ли? Если взять две быстровращающиеся нейтронные звезды, и сблизить их, так чтобы они не сколлапсировали, например пролетят на большой скорости, могут ли они друг другу сорвать оболочки + гравитацией уменьшить давление нейтронного газа и размотать в два гигантских джета? Или на таких расстояниях уже гравитация поведет себя странно и они все равно сколлапсируют?
Аноним 14/11/15 Суб 23:48:33 #157 №228796 
>>228789
Ну да, я не учел, что при прибавке вещества к звезде увеличится и количество возможных состояний, так что ты прав...
Насчет же двух нейтронок - конечно, нейтронки штука тяжелая, но всё ж не сингулярность. Коллапса в ЧД точно не будет при пролете, а вот срыв оболочек... невырожденную оболочку - запросто, а вот размотать вырожденный газ... Не знаю, если честно. Опять же, как их отличить от оболочки обычной плазмы? Хотя масса однозначно изменится, и можно будет уже по изменению массы констатировать "восстановление" нейтрония в плазму.
Аноним 15/11/15 Вск 11:52:07 #158 №228880 
>>227164 (OP)
Почему в портативной технике(смартфоны, ноктюрнбухтеры, ПЛОНШЕТКИ, камеры и т.д) используют литиевые говнобатареи, а не маленькие типа РИТЭГи, как в кардиостимуляторах? Почему именно литиевые, а не никелевые?
Аноним 15/11/15 Вск 12:17:32 #159 №228884 
>>228880
Первый раз слышу о ритэге в кардиостимуляторе думаю ты просто поехавший. Мысль абсурдная, т.к. ритэг генерирует тонны тепла и радиации, нуждается в толстом корпусе и т.д. Да и даже батарейка на радиоактивном изотопе не имплантировалась бы в сердце, лол. Там вроде обычные батарейки, просто прибор активируется только при проблеме с ритмом?
Литиевые без эффекта памяти.
Аноним 15/11/15 Вск 12:18:59 #160 №228885 
>>228884
Это он про Железного человека видимо))) Хотя не, там вроде термояд был)
Аноним 15/11/15 Вск 12:33:54 #161 №228889 
>>228885
Я бы посмотрел, как этот "гений, миллиардер, плейбой, филантроп" на ритэге бы летал. Вот например "легкий" ритэг, всего 600кг, ИЭУ-2М дает аж 20 Вт и 700 Ватт тепла, если предположить, что зависимость от весе/объема линейная, хотя это очень сомнительно, компактный ритэг айронмена, скажем легче в 300 раз - 2кг, но эффективнее раза в 3 дал бы ему аж 0,2Вт чего хватило бы например запитать светодиод на 15 лм! Даже если он совершил бы прорыв и использовал 100% тепловой мощности - для плутония это ~0,5 Вт на грамм, если не ошибаюсь, и запихал бы в себя скажем 1кг плутония и 1кг защиты и генератора - это всего 540Вт, лол, смог разве что бы сычевать с ПК в пустыне. И да, у него там термояд был.
sageАноним 15/11/15 Вск 12:43:11 #162 №228890 
>>228885
>>228884
Плутоний-236 и плутоний-238 применялся для изготовления атомных электрических батареек, срок службы которых достигает 5 и более лет. Их применяют в генераторах тока, стимулирующих работу сердца (кардиостимулятор)[8][9]. По состоянию на 2003 г. в США было 50—100 человек, имеющих плутониевый кардиостимулятор[10].
Аноним 15/11/15 Вск 13:02:49 #163 №228894 
>>228890
>50-100
>The team that carried out the first successful implantation in the U.S. developed a plutonium-powered pacemaker that would last for 30 years. But nuclear-powered pacemakers were impractical.
>These days, pacemakers last anywhere from about five to 12 years using lithium-iodine cell batteries.
Как оно, в 70х? Ссылка датирована 1974 в вики, откуда ты ее выдрал.
Аноним 15/11/15 Вск 14:16:56 #164 №228917 
14475862169730.jpg
>>228894
>Как оно, в 70х?
Хорошо. С 58-го РИТЭГи в кардиостимуляторах и он вполне себе маленький и работает. Что мешает установить аткой РИТЭГ в смартфон и получить смартфон работающий без подзарядки 5-20 лет? Что мешает установить РИТЭГ в электрокар/ядерный реактор с паровой турбиной ford nucleon
>>228880-кун
Аноним 15/11/15 Вск 14:18:19 #165 №228918 
>>228917
>С 58-го РИТЭГи используются в кардиостимуляторах и они вполне себе маленькие и работачие.
>такой
> Что мешает установить РИТЭГ в электрокар/ядерный реактор с паровой турбиной?
Самофикс
Аноним 15/11/15 Вск 14:34:26 #166 №228924 
Что такое больцмановский мозг?
Аноним 15/11/15 Вск 15:49:13 #167 №228930 
>>228917
>Что мешает
Неэкологишно и небезопасно.
Аноним 15/11/15 Вск 16:46:58 #168 №228940 
>>228917
> Что мешает
Принципиально - ничто. Если бы СССр был бы для людей а не против них, возможно мы сейчах и ходили бы с чугуневыми смаркфонами на плутонии или тории, и раз в нескольео лет сдавали бы их на диагностику и замену стекла/клавиатурки.

Но поскольку хозяевам денег невыгодно чтобы мобильные гаждеты были вечными, вот и приходится иметь дело с пласмасками/электроаккумами. Вон в ойфонах аккум даже несъемный был, никто революции не устраивал по этому поводу.
Аноним 15/11/15 Вск 18:10:54 #169 №228949 
>>228917
начинка ритега стала по три копейки пучек ?
миллионами тонн ?
безопасность по пизде ? будет смешно когда твоя бывшая намажет тебе трусы начинкой из этой хрени
вес на ватт ?
тебе мало ?
Аноним 15/11/15 Вск 18:12:26 #170 №228951 
>>228930
>Неэкологишно и небезопасно.
Гумус незаметен. АПАСНОСТЬ радиации сильно преувиличена.
http://www.citylabs.net/index.php?option=com_wrapper&view=wrapper&Itemid=25
Это, например, продаётся с 2012 года по цене несколько тысяч долларов/штука. В кардиостимуляторах плутоний-238, но ни у одного его "носителя" лучевой болезни не возникло.
Аноним 15/11/15 Вск 18:14:15 #171 №228952 
>>228949
>начинка ритега стала по три копейки пучек ?
Да, дорого
>миллионами тонн ?
Достаточно для того чтобы обеспечить тилибоны ею
>безопасность по пизде ? будет смешно когда твоя бывшая намажет тебе трусы начинкой из этой хрени
-> >>228951
И даже если намажет - ничего не будет
>вес на ватт ?
В кардиостимуляторах работает
>тебе мало
Да.
Аноним 15/11/15 Вск 19:09:25 #172 №228959 
>>228949
Стоп, а в кардиостимуляторах точно РИТЭГ? В смысле радиоизотопный термоэлектрический генератор? Или батарейка, то есть прибор, работающий на создании направленного движения электронов, без тепла?
Аноним 15/11/15 Вск 19:10:34 #173 №228960 
>>227164 (OP)
Что за f'sho?
Аноним 15/11/15 Вск 19:13:02 #174 №228961 
>>228960
"Наверняка", я так думаю. Ну или "без пи... брехни".
Аноним 15/11/15 Вск 20:34:48 #175 №228967 
>>228959
>в кардиостимуляторах
Термоэлектрический элемент на плутонии-238.
Т.е вполне себе РИТЭГ. А вот та "батарейка" на тритии - работает на электронах
Аноним 15/11/15 Вск 21:47:40 #176 №228991 
>>228952
про способы готовки плутония не буду, так как хз.

>вес на ватт ?
>В кардиостимуляторах работает

это не ответ, сколько кг рабочего вещества на кВт, вес целиком - ты же в машины хотел пихать. (если тебе лень гуглить ты можешь посчитать, мне с точностью до грамма не надо)

>>228951
>Гумус незаметен. АПАСНОСТЬ радиации сильно преувиличена.
я бы не стал так над чуваком прикалываться, собственно пиздеть, о преувеличенности опасности радиации, ты можешь в том числе и благодаря именно таким нервным чувакам, а не ходить почесывая жопу третьей рукой. Тут уже были истории как чуваки разбирали ритеги, и всякий прочий криминал с духе спиздил и нашел, а оказалось не то ?
Нахуй нахуй короче - с таким процентным соотношением дебилов это нахер не надо.
Анон 16/11/15 Пнд 02:13:36 #177 №229056 
14476292164600.jpg
Ананасы, а что измениться в физике, если бы скорость света не была бы непреодолимой? как я понимаю как раз изза конечности скорости света (точнее изза того, что нельзя двигаться со скоростью большей скорости света) и возникают всякие интересные штуки, вроде замедления времени и искривления пространства и так далее.

Какие изменения бы произошли в законах природы и теориях относительности?
Аноним 16/11/15 Пнд 06:15:02 #178 №229069 
Если нашу Землю зашвырнуть в глубокий космос, её ядро остынет и затвердеет (через какое-то время)?
Аноним 16/11/15 Пнд 07:34:36 #179 №229072 
>>229069
Через 10 миллиардов лет вполне возможно.
Аноним 16/11/15 Пнд 07:35:32 #180 №229073 
>>229056
Но скорость света не является конечной.
Аноним 16/11/15 Пнд 08:12:35 #181 №229075 
>>228991
Для плутония - 0.57 Вт тепловой энергии/г.
Аноним 16/11/15 Пнд 08:54:25 #182 №229081 
>>229069
Оно и так застынет и отвердеет по идее, если раньше не окажется внутри раздувшегося до красного гиганта Солнца.
Аноним 16/11/15 Пнд 08:57:35 #183 №229082 
Заранее извиняюсь, тред не читал, возможно, такой вопрос уже звучал.
Правильно я понимаю, в данный момент Вселенная расширяется на остаточной энергии от Большого Взрыва. Рано или поздно, но она закончится и вещество с периферии начнёт притягиваться веществом с центра и так, по цепной реакции, всё сожмётся в одну точку, произойдёт новый Большой Взрыв и т.д., т.е. это обычный цикл развития вселенной. Возможно, дикую хуйню сейчас сморозил, так что поправьте.
Аноним 16/11/15 Пнд 09:25:47 #184 №229085 
>>228951
>АПАСНОСТЬ радиации сильно преувиличена.
А вот на вики написано, что после падения совка охотники за цветметом в том числе и на земляные ритеги совершали набеги, унося в теле смертельное число рентген.
Аноним 16/11/15 Пнд 09:43:47 #185 №229086 
>>229082
бамп вопросу
мимо
Аноним 16/11/15 Пнд 09:45:16 #186 №229087 
14476563163990.jpg
>>229082
Нет, хуйня не дикая и уже рассмотренная учеными мужами с нескольких ракурсов. Суть в том, что современная методология не позволяет сформулировать фальсифицируемую, т. е. научную теорию о циклах жизни вселенной.

Методология предполагает эмпирическое наблюдение за средней плотностью вселенной и на основе этих сугубо умозрительных данных получение выводов о перспективах. Дуализм перспектив заключается в "большом разрыве" и "большом сжатии". Либо вселенная сожмется в новую единую сингулярность под действием гравитационных сил, либо расширение пространства приобретет грандиозные масштабы и все четыре фундаментальных взаимодействия не смогут удерживать материю в привычной нынче форме. На данный момент, насколько я помню, данные говорят о большей вероятности разрыва, нежели сжатия, но нужно гуглить.
Ах да, расширение пространства - его исключительное свойство, объяснить которое еще предстоит, потому что природа темной энергии также темна, как и сама энергия. Не стоит представлять взрыв как некий упругий удар сжавшейся материи.
Аноним 16/11/15 Пнд 10:32:25 #187 №229089 
>>229056
>что измениться в физике, если бы скорость света не была бы непреодолимой?
Если просто выше световой (но конечной) - то серьезно пострадали бы многие научные теории, ну а так - ничего особенного.

А вот если бы она стала бесконечной... Это уже даже не физика. Нарушился бы принцип причинности, и, вероятно, это привело бы к "распаду" всего.
Аноним 16/11/15 Пнд 10:39:19 #188 №229090 
Космокрест даун ИТТ. Уже и хейтерский тред найти не могу.
Новости есть какие?
Аноним 16/11/15 Пнд 10:51:38 #189 №229092 
14476602986390.jpg
>>229087
Зачем ты здесь методологию науки приплел - вообще непонятно. Очень трудно дать прогноз на столь длительное время. Насколько помню (доминирующая сейчас) Лямбда-CDM-модель предполагает и дальнейшее увеличение постоянной Хаббла, всё большее растяжение пространства, и, в конце-концов, наступление Эпохи Тьмы (ололо; смайлик).

Совсем простым языком:
https://ru.wikipedia.org/wiki/Будущее_Вселенной
Аноним 16/11/15 Пнд 11:36:08 #190 №229095 

>>229082
Человек выше прав, но еще добавлю - этот большой разрыв грозит только при ныне наблюдаемой плотности. Не исключено, что мы проморгали сотни массы (из-за того, что она не светится) и тогда нам всё же грозит либо остановка расширения, либо сжатие.
Аноним 16/11/15 Пнд 11:53:33 #191 №229099 
>>229092
>Зачем ты здесь методологию науки приплел - вообще непонятно
Пиздец ты формалист. Как насчет чтобы пояснить невозможность однозначного ответа на поставленный вопрос?
Аноним 16/11/15 Пнд 12:00:42 #192 №229102 
>>229099
Я попробую пояснить. В науке есть такой метод, как экстраполяция. То есть, наблюдаешь некоторое время процесс, набираешь его статистику, а потом на основе этой статистики пытаешься предсказать поведение процесса в дальнейшем. Так вот. Чем большее время занимает наблюдение - тем точнее прогноз. Теперь посмотрим на Метагалактику. Она расширяется около 15 млрд лет. Мы этот процесс наблюдаем меньше 100 лет. И все современные теории позволяют применять для прогнозирования в основном только экстраполяцию. Ну и откуда взять однозначность?
Нужно либо изменить метод изучения расширения, либо пронаблюдать за расширением пару млрд лет, тогда можно будет говорить о точности.
Аноним 16/11/15 Пнд 12:04:34 #193 №229103 
>>229102
Толково. Даже я понял. Спс за разъяснение, анон.
Аноним 16/11/15 Пнд 12:35:34 #194 №229118 
Ну всё, тусовочка эфирщиков собралась. Про топологию они знают только название, но гордо бьют себя айфоном в грудь: "я не поддамся на их сказки!!!11"
Аноним 16/11/15 Пнд 12:57:55 #195 №229125 
>>229102
Бомбанул и не понял. Зачем ты мне за экстраполяцию поясняешь? Я тебе намекаю, что описание методологии в посте>>229087 вполне уместно, потому что демонстрирует ее поверхностность и несостоятельность в вопросе долгосрочных предсказаний, причиной которых как раз и являются слишком малые сроки наблюдений.
Аноним 16/11/15 Пнд 13:35:14 #196 №229138 
>>229073
Почему? Скорость света это константа.
Аноним 16/11/15 Пнд 14:00:18 #197 №229140 
>>229095
Я не очень понимаю почему ученые решили что есть темная материя, ведь они не могут увидеть и посчитать всю материю нашей галактики, однако заявляют что её недостаточно чтобы она вертелась так как вертится. Может быть она есть, и её много, просто её не видно.
Аноним 16/11/15 Пнд 14:35:10 #198 №229142 
>>229140
Ты не поверишь, но темная материя и есть та, которую не видно. На ее место есть несколько "несветящих" претендентов. Это вовсе не магическая структура.
Аноним 16/11/15 Пнд 14:49:51 #199 №229146 
>>229142
Но ведь ученые говорят что темная материя состоит не из привычных нам барионов. Я раньше думал что экзопланеты это тоже тёмная материя, но кажется так думать не правильно.
Аноним 16/11/15 Пнд 15:29:35 #200 №229159 
>>229146
Представь, ты по комнате идешь в темноте, а там два негра мазутом обмазались и ебутся. Ничего сверхъестественного, просто их вообще не видно, зато слышно отлично. Ты знаешь что они есть, просто не можешь посмотреть на лица.
Так и тут, вычислили что есть скрытая масса эти двое, но они совершенно не видны в темноте. Вот и строят предположения почему, мазутом обмазались и сами черные, и откуда взялись. Барионная материя плохо подходит, вот гадают над небарионной. Повторю - магии видимо никакой нет, просто лиц не видно.
Аноним 16/11/15 Пнд 15:37:34 #201 №229166 
>>229140
>ведь они не могут увидеть и посчитать всю материю нашей галактики
Гравитирующую массу. По воздействию гравитации (и других сил) эту материю детектят. Расхождение с законом Вина ("вириальный парадокс") и дает, во многом, данные о скрытой массе.
Аноним 16/11/15 Пнд 18:32:41 #202 №229203 
>>229075
неплохо, я думал хуже будет
Аноним 16/11/15 Пнд 18:35:23 #203 №229204 
>>228951
тем более линка на тритий
> В процессе распада тритий превращается в 3He с испусканием электрона и антинейтрино (бета-распад), период полураспада — 12,32 года. Доступная энергия распада очень мала (18,59 кэВ), средняя энергия электронов 5,7 кэВ.
Аноним 16/11/15 Пнд 18:59:42 #204 №229211 
мочератор, какого лешего, зачем закрыл тред про биосферы.
Проблема замкнутых биологических систем применительно к космасу есть, а треда нет, и это тогда когда я хотел написать ответ что осваиваемость океаносферы на данный момент не такая незначительная, хорошо я по скромному не стал тратить час на сбор материалов по данному вопросу, но все равно.

То что тред стартанул не очень, это конечно так, но в принципе аноны делают тред, а не тред анонов, в конце концов.
мне стало легче, спасибо
Аноним 16/11/15 Пнд 19:41:36 #205 №229218 
>>229085
В совковых недоритэгах стронций-90
Аноним 17/11/15 Втр 06:45:03 #206 №229320 
>>227164 (OP)

Пейсаны, из какого фильма срезали этот кусок?
https://www.youtube.com/watch?v=pD-pmLHAiGA
Аноним 17/11/15 Втр 06:47:05 #207 №229321 
>>229089
> бесконечной
Я имею ввиду не скорость распространения фотонов, а скорость, которой нельзя достигнуть барионной материи. Например что было бы если бы на ракете можно было бы бесконечно долго ускоряться и превысить скорость света?

> то серьезно пострадали бы многие научные теории
Вот как раз это меня и интересует, давай по-конкретнее что там пострадало бы.

Аноним 17/11/15 Втр 07:52:10 #208 №229323 
>>229321
Ты можешь облететь всю вселенную за секунду, но для вселенной это не будет секундой. Никакие законы физики не нарушатся. Главная проблема состоит в нахождении иного пути ускорения материи.
Аноним 17/11/15 Втр 08:58:55 #209 №229327 
>>229321
>скорость, которой нельзя достигнуть барионной материи. Например что было бы если бы на ракете можно было бы бесконечно долго ускоряться и превысить скорость света?
>Вот как раз это меня и интересует, давай по-конкретнее что там пострадало бы.
Да хоть законы сохранения (из-за лореренцевых преобразований). В какой-то момент придется тырить массу-энергию у алиенов из соседней вселеняшки. При бесконечном полете - это уже философия, а не физика.
Аноним 17/11/15 Втр 10:01:35 #210 №229335 
>>227654
гугли ИМБП
Аноним 17/11/15 Втр 10:11:03 #211 №229337 
>>227164 (OP)
Если на луне взорвать бомбу на 50 мегатонн, могут ли камни долететь до земли? Если да, то много ли?
Аноним 17/11/15 Втр 10:33:54 #212 №229340 
14477456349220.jpg
>>229337
Чтобы только покинуть орбиту Луны нужна вторая космическая скорость, которая для Луны составляет 2 400 м/с или около того. Далее нужно проводить довольно массивные расчеты, учитывающие бризантности и фугасности конкретного взрывчатого вещества, но я наугад отвечу, что вероятней всего энергии таки не хватит.

Являясь полным профаном в вопросах саперного дела и прочей пиротехники, предположу, что взрывчатка не сможет передать скорость телу большую, чем скорость детонации. Оная для самых лучших образцов ВВ не превышает 9 км/с. Даже если гипотетическая песчинка полетит в сторону земли с первоначальной скоростью в 9 км/с, то лунная гравитация каждую секунду будет отбирать по полтора метра из этой скорости. Да, чем дальше будет лететь песчинка, тем меньше будет притяжение, но я все равно считаю, что покинуть область притяжения Луны она не успеет.
Олсо мощность взрыва тут особенного значения не имеет, если это не мощность, способная уничтожить луну. Для конкретной цели нужна скорее инженерная конструкция, создающая направленный взрыв, нежели мощная боньба. И да, ядерный взрыв для цели создания сейсмической волны подходит хуже, потому что материя в эпицентре взрыва будет просто испарятся, и ударная волна соответственно ослаблена. Взрывать же бомбу на поверхности и вовсе нет смысла, потому что ввиду отсутствия атмосферы, большая часть энергии взрыва уйдет в грунт и в излучение.
Аноним 17/11/15 Втр 10:40:00 #213 №229342 
>>229340
Добавлю еще кое-что. Помню, из-за всех этих обстоятельств я классе эдак в 11-м дико смеялся над Жюль-Верновским "Из пушки на Луну". Конеееечно, они пироксилином добросили снаряд до Луны))) Там же скорость сгорания вообще ниже плинтуса...
Аноним 17/11/15 Втр 10:53:50 #214 №229344 
>>229337
Хуй знает, но доказано, что на землю прилетают куски с Луны и даже Марса, отколовшиеся в результате падения метеоритов.
Аноним 17/11/15 Втр 11:08:58 #215 №229348 
>>229344
Есть версия (не знаю, доказана ли) что в космос булыжник может запульнуть и вулкан. А у метеорита нет такого понятия, как "скорость детонации". Энергия удара определяется массой и скоростью, а летают они ОЧЕНЬ быстро.
Аноним 17/11/15 Втр 14:02:19 #216 №229435 
14477581395380.gif
>>229337
>>229340
на мунспике нет, только так https://en.wikipedia.org/wiki/Operation_Plumbbob#Propulsion_of_steel_plate_cap

>>229342
если воспринимать эту идею на уровне концепта, то при определенной доле модификации она вполне годна, хотя она и выглядит после этого как hyperloop, но дедушка Жуль был норм.
Аноним 17/11/15 Втр 14:25:03 #217 №229436 
>>229435
Опиши вкратце, #чё_там_у_Plumbbob, а то лень в детали вникать.
Аноним 17/11/15 Втр 15:28:33 #218 №229448 
14477633136520.jpg
Почему безмассовые бозоны имеют энергию, но не имеют массы? так такое вообще может быть? Энергия этож и есть масса, объясните пожалуйста.
Аноним 17/11/15 Втр 15:30:34 #219 №229449 
>>229448
Ты сейчас о чем? Конкретнее, батенька.
Аноним 17/11/15 Втр 15:33:43 #220 №229450 
>>229448
Есть две массы - масса покоя и просто масса. Вот массы покоя у них и нету, а вот простая - как сумма массы покоя и "скоростной" - есть.
Аноним 17/11/15 Втр 15:44:44 #221 №229455 
>>229450
чем масса покоя от простой массы отличается?

и если проста я масса = масса покоя + скоростная, то что еще за скоростная?
Аноним 17/11/15 Втр 15:48:45 #222 №229456 
>>229455
Тело, при увеличении скорости, увеличивает и массу.
Аноним 17/11/15 Втр 15:52:47 #223 №229459 
>>229455
Анон выше прав, только у некоторых частиц есть масса даже при нулевой скорости - это и есть масса покоя. А есть еще и добавка от скорости - они с массой покоя и образуют полную массу.
Аноним 17/11/15 Втр 15:56:42 #224 №229461 
>>229459
ну как я понимаю у фотона отсутствует масса покоя, это значит, что для того, чтобы остановить не поглощая - нужно приложить бесконечное количество энергии?
Аноним 17/11/15 Втр 16:03:31 #225 №229462 
>>229461
Скорее остановленный он не имеет смысла, т.е. просто не может существовать на любых скоростях, кроме скорости света.
Аноним 17/11/15 Втр 16:12:50 #226 №229463 
Где то читал, что объект может пересечь горизонт событий чёрной дыры. Разве это не пиздёшь? Если с точки зрения внешнего наблюдателя время объекта остановится на горизонте событий, то с точки зрения самого объекта время вселенной бесконечно ускорится. Если верить Хокингу, ЧД не существуют вечно. Разве это не значит, что с точки зрения затягиваемого объекта, ЧД сдуется раньше, чем он достигнет горизонта?
Аноним 17/11/15 Втр 16:14:47 #227 №229464 
14477660878160.png
>>229461
Это не так. Я имел в виду обычное тело, обладающее массой покоя. "Масса покоя" фотона равна пикрилейтед, хотя и это не совсем верно. Внутренняя четность частицы не определена, и не существует системы отсчета, относительно которой эта частица покоится. Ну и там еще три тома муры и всякого матана.
Аноним 17/11/15 Втр 16:41:36 #228 №229471 
>>229463
Это интересная мысль. Однако падающий объект не достигнет скорости света, на горизонте скорость падения не будет равна световой, поэтому время ускорится не бесконечно. А вот за горизонтом...
Аноним 17/11/15 Втр 16:50:52 #229 №229474 
>>229471
А за горизонтом - уже какая-то другая физика со своими Эйнштейном, пространством и прочим) Так что там пусть разгоняется хоть до миллиона световых - никакие законы не будут нарушены)
Аноним 17/11/15 Втр 16:53:33 #230 №229475 
>>229436
накрыли шахту крышкой, ебнули бонбу, крышку сорвало, болшее ее никто не видел
гадают действительно ли ее разогнало до 66км/c и вышла ли она на орбиту солнца или испарилась к ебеням при прохождении атмосферы, когда ее последний раз видели (один кадр на рапиде) то она все еще была вполне крышкой, а не облаком газов.
Аноним 17/11/15 Втр 20:20:16 #231 №229506 
>>229435
Ну, да. Получается если на Луне прокопать лунку, сунуть в нее боньбу и создать в канале условия для образования взрывной волны, то железными люками можно бомбардировать Землю, ибо лунная атмосфера стереть люк все же не сможет. Вот только до поверхности земли эти люки тоже не долетят.
Аноним 17/11/15 Втр 20:51:01 #232 №229541 
14477826618560.png
>>229506
ну метеориты долетают, такчто и от люков никто не застрахован
Аноним 18/11/15 Срд 02:48:10 #233 №229687 
Как выглядит более подробно, планетарная система из к\ф Чёрная Дыра? Насколько возможна данная система?
Аноним 18/11/15 Срд 02:53:57 #234 №229689 
14478044371260.jpg
А почему ГГ из интерсталера проходя через гиперпространство, не встретил вашу-чан?
Аноним 18/11/15 Срд 07:54:14 #235 №229698 
>>228094
А почему невозможна энергетическая жизнь ?
Аноним 18/11/15 Срд 09:11:47 #236 №229702 
>>229698
Почему ты думаешь, что она невозможна? Мы не можем даже предположить на каких принципах она бы строилась, поэтому научных представлений нет. Хотя почему бы жизни не существовать в плазме? Есть ли жизнь в плазме, нет ли жизни в плазме. Ответ на этот вопрос науке неизвестен.
Аноним 18/11/15 Срд 17:51:08 #237 №229845 
>>229702
> Хотя почему бы жизни не существовать в плазме?
Для жизни нужна сложная устойчивая структура. Плазма слишком нестабильна.
Аноним 18/11/15 Срд 17:55:29 #238 №229847 
что там с темной материей и энергией, чтото меня пробило с того что 95 процентов вселенной хуй знает что есть.
по хардкору - есть шанс что я узнаю что это до того как вселенная обратится в ничто?
Аноним 18/11/15 Срд 18:25:18 #239 №229861 
>>229845
Посмотрите-ка, обсуждаем углеродный шовинизм, его защищает углеродный шовинист. Я написал, что сейчас иных представлений о жизни, кроме нашей (и близкой) у нас нет, но это не значит, что они не могут быть. Вот тебе желтая статейка на тему, чтобы понимать о чем я вообще: http://www.membrana.ru/particle/693 .
Аноним 18/11/15 Срд 22:19:38 #240 №229983 
>>227164 (OP)
А что если сейсмоактивность на Плутоне является результатом воздействия темной энергии на него.
Аноним 18/11/15 Срд 22:22:31 #241 №229986 
>>229847
А что с тобой станет когда узнаешь, что ты состоишь на 99.9% из пустоты?
Аноним 19/11/15 Чтв 00:53:08 #242 №230023 
>>229986
хм ну это филосовский вопрос, я как бы в курсе, но с другой стороны я и о темной материи черной энергии был в курсе, такчто ответ на твой вопрос undef
вообщем чето торкнуло как того чувака с юпитера, спасите нас темные боги хаоса от таких приходов
Аноним 19/11/15 Чтв 13:41:15 #243 №230094 
Где находится черная материя/энергия если ее нету в солнечной системе? Между звездными системами, между галактиками? Вообще чем отличается космос возле планет и звезд от космоса между скоплениями и галактиками, ну кроме излучения звезды: ветра там, гравитации звезд.
Аноним 19/11/15 Чтв 13:54:54 #244 №230103 
>>230094
>если ее нету в солнечной системе?
C чего ты решил, что ее в СС нет?

>Между звездными системами, между галактиками?
Предполагают, что в основном темная материя концентрируется в гало галактик. Ну а темная энергия - просто свойство пространства. Пока, правда, о ней почти ничего не известно.
Аноним 19/11/15 Чтв 14:32:39 #245 №230119 
Анон, не пояснишь "на пальцах" гуманитарию суть эффекта Оберта?
Читал статью в Википедии — нихрена не понял. Ну движется ракета быстрее, ну выше у топлива кинетическая энергия — но почему ж дельта-V разная?
Вот есть движок, который ускоряет сферического коня в вакууме на 10 км/ч за минуту. А из-за эффекта Оберта, получается, при достижении высоких скоростей тот же двигатель при тех же затратах топлива будет ускорять коня уже на 20 км/ч?
Какой-то ИРЛ-чит получается.
Аноним 19/11/15 Чтв 18:18:21 #246 №230216 
>>230119
В Википедии, как часто бывает, очень плохое объяснение. Я сейчас тоже плохо объясню, но надеюсь всё же чуть понятнее.

>Вот есть движок, который ускоряет сферического коня в вакууме на 10 км/ч за минуту. А из-за эффекта Оберта, получается, при достижении высоких скоростей тот же двигатель при тех же затратах топлива будет ускорять коня уже на 20 км/ч?

Не так. Прирост скорости останется всё теми же 10 км/ч. Фишка в том, что кинетическая энергия растёт с квадратом от скорости. Поэтому чем больше твоя начальная скорость, тем больше кинетической энергии ты получишь от того же увеличения скорости.

Пример в условных единицах:

1) начальная скорость 10, прибавляем 10. Получаем (10+10)^2 = 400 (прирост 300).

2) начальная скорость 100, добавляем 10. Получаем (100+10)^2 = 12100 (прирост 2100)

Ключевая часть -- именно вот этот квадрат.
Аноним 19/11/15 Чтв 19:37:21 #247 №230241 
>>230216
Эм... Но ведь это же справедливо для любой телеги с мотором (или без)?
Аноним 19/11/15 Чтв 21:20:36 #248 №230267 
>>230119
есть ракета скорость ее на нуле, скорость истечения газов 4кмс к прмеру. Стоишь ты рядом, ракета зафырчала и в первый момент не быстро так поехала. С твоей точки зрения основная масса импульса профукана в скорость газов которые мимо тебя улетели.
пролетает ракета мимо тебя на скорости 4кмс, реактивная масса относительно твоей точки зрения имеет скорость ноль, и можно сказать что с твоей точки зрения полезность работы реактивной массы выше чем в первом случае. Скорость оно такое понятие относительное с точки зрения инерциальных систем отсчета, коей ракета не является по определению, но наблюдатель и реактивная масса да(при определенных условиях). но хз я может и не о том, просто вариант посмотреть на эту проблему с такой точки зрения
Аноним 20/11/15 Птн 00:48:05 #249 №230310 
>>227164 (OP)
Приснился ебанутый сон. По его результатам назрело пара вопросов:

Что будет если Земля будет вращаться гораздо быстрей? Например если на оборот вокруг солнца будет уходить секунд 5?

И что будет если с такой же скоростью вокруг Земли будет вращаться луна.
Аноним 20/11/15 Птн 01:02:10 #250 №230314 
>>230310
Земля улетит в открытый космос.
Луна улетит в открытый космос.
Аноним 20/11/15 Птн 01:29:15 #251 №230329 
>>230119
Его суть в том, что за одно и тоже количество потраченного топлива можно наверно даже сказать "потраченной dV", ты получаешь больше орбитальной скорости = кинетической энергии. Можно считать что когда ты находишься на эллиптической орбите Земли, ее орбиты полная энергия это высота орбиты (потенциальная энергия) и орбитальная скорость (кинетическая энергия). Например, перицентр - это место, где ты имеешь наибольшую кинетическую энергию и наименьшую потенциальную - наиболее выгодное для маневра. Здесь ты и хочешь включить двигатель. Во время его работы ты выбрасываешь за борт массу, от которой сам и отталкиваешься, обмениваясь с ней кинетической энергией. Таким образом никакого читерства, ты просто отбираешь побольше кинетической энергии/орбитальной скорости у выхлопа.
Аноним 20/11/15 Птн 01:35:59 #252 №230331 
>>230310
Если вдруг, по какой-то случайности, Земля при этом и не улетит искать другое солнце, то улетит всё с ее ночной поверхности - ибо вращательное движение по определению ускоренное. То есть ускорение на стороне, обращенной в ночь, будет направлено против притяжения Земли и будет превышать ее же 2-ю космическую скорость. Так что это будет звиздец. И да, если тебя ночью не унесет в космос - днем тебя расплющит, так как вектор центробежного ускорения будет направлен в землю.
Аноним 20/11/15 Птн 06:15:46 #253 №230347 
Два тупых вопроса:
1. Какими свойствами обладает антиматерия? Например брусок железа отлитый из античастиц, будет ни чем не отличатся от обычного? Или же он будет обладать другими свойствами?
2. Предположим у нас есть 100 кг антиматерии в магнитной ловушке, что будет если взять человека и кинуть его туда? Он умрет-аннигилирует? Как быстро?
Аноним 20/11/15 Птн 09:24:35 #254 №230357 
>>230331
>обращенной в ночь
Чего блять? Центробежная сила - сова дохуя? Какая нахуй ночь?
>>230310
У тебя условия нарушают законы физики. Луна не может иметь такую орбитальную скорость в первую очередь по причине того, что ее орбита имеет длину около 7 световых секунд, а во вторых происходит явное нарушение принципов орбитальной механики. Тело не может вращаться с такой скоростью, потому что притяжение земли не даст уравновешивающего для вращения нормального ускорения.
А вращение земли с такой скоростью увеличит центробежную силу в ~1 000 000 раз и гравитация планеты не сможет удерживать материю. Я попытался было найти наглядное видево с центрифугой, но не смог.
Аноним 20/11/15 Птн 09:26:46 #255 №230359 
>>230331
Каюсь, нагрубил спросонья. Думал он про 5 секунд вращения вокруг орбиты спрашивает.
Аноним 20/11/15 Птн 09:41:28 #256 №230361 
>>230347
Это уже материаловедение. Трудно сказать, какими свойствами будет обладать такой кусок антижелеза (если его вообще удастся сформировать). Хотя, может кто производил теор.изыскания по сопромату антиметаллов.

И да, при контакте с антивеществом обычный анон-пряник "умрет-аннигилирует". Быстрее миллионных долей секунды все 100 кг антиговна прореагируют.
Аноним 20/11/15 Птн 10:19:10 #257 №230366 
Видел передачку ббс про край вселенной, там сказали что если улететь достаточно далеко, то можно поймать первую радиопередачу в мире. Радио это ведь волны определенной частоты, исходя из передачи выходит, что этот сигнал по сути вечен, тоесть летит он себе и летит. В связи с этим два вопроса, первым задам особо тупой:
1. Когда человек говорит, он ведь ро сути испускает звуковые волны? Если да, то все что мы говорим вечно? Как поймать эти волны?
2. Опять же исходя из передачи выходит, что весь космос завален волно-хламом, ведь сколькл человечество волн напускало радио, теле и вот это все?
Аноним 20/11/15 Птн 10:22:02 #258 №230367 
>>230366
>если улететь достаточно далеко, то можно поймать первую радиопередачу в мире
Это ж с какой скоростью надо лететь?
Аноним 20/11/15 Птн 10:32:17 #259 №230370 
>>230367
Спроси деятелей из ббс. Я человек простой - вижу тред тупых вопросов, захожу, туплю.
Аноним 20/11/15 Птн 10:34:56 #260 №230372 
>>230366
Свет от звезд, и еще невероятное множество частот испускается разными объектами во вселенной. Все это такой же электромагнитный хлам. Который перемешивается, интерферирует и превращается из информации в кашу. Чем дольше летит волна, тем меньше она соответствует заложенной в ней информации. И с учетом мощностей галактических источников, вклад человечества в общий волновой воспринимать как хоть сколько-нибудь влияющий не стоит.
>>230366
>Если да, то все что мы говорим вечно?
Радиоволны по сути управляются одним из фундаментальных взаимодействий. На колебания воздуха же влияет куча других макрофизических факторов, основной - сопротивление среды, которое гасит со временем звуковые волны. Нет, они не вечны.
Аноним 20/11/15 Птн 10:40:41 #261 №230375 
>>230372
Грац за ответ. Я правильно понял: колебания воздуха это не волны, когда мы говорим это колебания воздуха? Если да, то можно разницу вкратце, смутило что волны это ведь тоже колебания, только не воздуха наверное.
Аноним 20/11/15 Птн 10:45:29 #262 №230378 
>>230375
Колебания и есть волны, что воздушные, что электромагнитные. Они подчиняются общим законам и имемют соответствующие параметры частоты, амплитуды и т. д. Но природа разная.
Аноним 20/11/15 Птн 10:48:35 #263 №230379 
>>230378
Электромагнитные лучше сохраняют и переносят да?
Аноним 20/11/15 Птн 11:01:07 #264 №230380 
>>230379
Что сохраняют, ничего они не сохраняют. Просто электромагнитная волна - базис, фундаментальный элемент. А всякая материальная среда на земле - комплекс материи и фундаментальных взаимодействий. Поэтому волна может изменить свои первоначальные параметры до неузнаваемости, но она останется волной, потому что это базис. Колебания воздуха могут изменится во множество вещей, энергия волны может быть передана любому другому объекту и потерять всякую первичную структуру. Поэтому электромагнитные волны летят миллиарды лет, а колебания воздуха угасают в промежутке секунд/минут
Аноним 20/11/15 Птн 11:04:20 #265 №230381 
Ну выше написано "все меньше соответствуют заложенное в них информации", а теперь ты говоришь что они ничего не сохраняют, так в них есть что оо или нет блеат?
Аноним 20/11/15 Птн 11:04:30 #266 №230382 
>>227503
Я такую в пионерском лагере запускал. Милота.
Но самая пиздатая получилась из спижженого на стройке карбида и бутылки из под шампанского.
Аноним 20/11/15 Птн 11:07:55 #267 №230385 
>>230381
Ты понимаешь что такое "интерференция"? Кончено эм-волны "лучше" сохраняют. Голосом ты не можешь дальше полукилометра общаться, а радиоволнами управляют марсоходом. Но вот это понятие "лучше" в масштабах вселенной слишком незначительно, чтобы воспринимать эм-волны как "вечные".
Аноним 20/11/15 Птн 18:05:50 #268 №230517 
Посоны, в этом видосе же рассказывают про какие-то гипотетические возможности в основном белые дыры, множественные вселенные? Или это уже теоретически обосновано?

https://www.youtube.com/watch?v=Qn4VooVCcrM
Аноним 20/11/15 Птн 18:06:29 #269 №230518 
>>230517
Блять, вместо спойлера болд захуярил, ну ладно.
Аноним 20/11/15 Птн 18:11:07 #270 №230519 
>>230517
>белые дыры, множественные вселенные
Видео не смотрел, а весь твой болд - хуита.
Аноним 20/11/15 Птн 22:58:30 #271 №230651 
Зачем нужна еденица измерения зиллиард?
Аноним 21/11/15 Суб 09:59:31 #272 №230765 
Почему газы из камеры сгорания вылетают только в сопло, а не обратно в ТНА (со стороны которого давление естественно меньше, чем внутри камеры)?
Аноним 21/11/15 Суб 12:48:36 #273 №230792 
>>230651
https://www.youtube.com/watch?v=TUagt12CFuQ
Низачем. Её нет.
Аноним 21/11/15 Суб 14:17:51 #274 №230803 
14481046717670.png
>>230792
Помню была на вики. Ну ладно, зачем вот эти например?
Аноним 21/11/15 Суб 15:48:33 #275 №230813 
>>230803
А зачем Юпитер?
Они просто есть. Надо же их назвать как-то.
Аноним 21/11/15 Суб 15:51:37 #276 №230815 
test
Аноним 21/11/15 Суб 16:01:55 #277 №230816 
>>230813
Но Юпитер не был придуман человеком, он был всегда. А математику как раз таки придумали.
Аноним 21/11/15 Суб 16:27:45 #278 №230818 
>>230816
А это не важно, был всегда или появился, когда его увидели.
Важно, что в обеих системах (ИРЛ и математика) имеются вещи, которые не были в них, кхм, "предусмотрены" изначально, но которые, тем не менее в них существуют. Едва ли древние счетоводы задумывались о таких ебических числах. Но в изобретённой ими системе эти числа вполне себе существуют, неважно, используются они на практике или нет. Быть может, число 112847412831209138419 было только что написано впервые в истории человечества - но оно же должно как-то называться, чтобы была возможность сообщить о нём? Хотя бы потому, что ненормальна ситуация, когда вещь есть, а термина для неё нет.
Странный вопрос, короче.
Аноним 21/11/15 Суб 16:33:22 #279 №230819 
В общем, тред скатился к "жопа есть, а слова нет".
Аноним 21/11/15 Суб 16:36:30 #280 №230822 
>>230803
Чтобы описывать размер моего хуя.
Аноним 21/11/15 Суб 16:55:43 #281 №230830 
Когда станет известно что было до большого взрыва? Сможет ли человеческий мозг понять это, ведь он до сих пор понимает бесконечность как что-то просто вечно расширяющиеся?
Аноним 21/11/15 Суб 16:57:46 #282 №230833 
>>230830
>Когда станет известно что было до большого взрыва?
Завтра в 15:32. Телек включай, там скажут.
Аноним 21/11/15 Суб 17:26:15 #283 №230848 
14481159754450.jpg
>>230830
Я откуда знаю?
Аноним 21/11/15 Суб 20:36:02 #284 №230911 
>>230765
>со стороны которого давление естественно меньше, чем внутри камеры
Нет.
Аноним 21/11/15 Суб 20:59:25 #285 №230921 
>>230911
Тогда зачем вообще поджигать топливо?
Аноним 21/11/15 Суб 21:35:04 #286 №230946 
>>230921
Чтобы использовать запасенную в нем химическую энергию.
Аноним 21/11/15 Суб 21:41:02 #287 №230950 
>>230921
Чтобы вместо тонкой струйки, ссущей из топливного насоса, на выходе из сопла ебошил поток продуктов сгорания, например.
Аноним 21/11/15 Суб 21:55:12 #288 №230955 
14481321128530.png
>>230818
походу в натуре
Аноним 21/11/15 Суб 22:16:28 #289 №230965 
Спейсач такой вопрос, что мешает втихаря забросить на разные орбиты несколько "облаков" гаек и болтов общей массой до 25 тонн, с целью выпила всего парка спутников, кроме высотных на геостационаре. Как лучше всего пускать?, какие орбиты самые "эффективные" для этих целей? Каков разброс облака должен быть? Каковы последствия?
Аноним 21/11/15 Суб 22:21:56 #290 №230968 
>>230965
>что мешает
Здравый смысл.
Аноним 21/11/15 Суб 22:32:44 #291 №230981 
>>230968
ну давай чисто пофантазируем. Вот наводнили спутники шпионы небо, куча всяких китайских индуских станций понаразвелось. А тут такую МКС списывают. Просто топить неинтересно. и вот мы выводим эти 25 тонн на круговую? и небольшим зарядом распыляем их на высоте 410 км, что происходит дальше? Насколько быстро облако мусора вырастет? насколько быстро начнет тормозится и спускаться ниже?
Аноним 21/11/15 Суб 22:40:19 #292 №230985 
14481348193920.jpg
Поиграл в SOMA и возник вопрос — можно ли теоретически построить огромную гаусс-пушку, которая выстрелит снарядом на околоземную орбиту?
http://soma.wikia.com/wiki/Omega_Space_Gun
Аноним 21/11/15 Суб 22:47:10 #293 №230991 
>>230965
> Как лучше всего пускать?
чтоб не засекли
>какие орбиты самые "эффективные" для этих целей?
полярные орбиты естественно

>>230981
разойдется достаточно быстро, тормозить будет очень медленно
Аноним 21/11/15 Суб 22:50:06 #294 №230994 
>>230985
можно, но дохуя большая будет, в первую очередь потому что если выстрелить просто болванкой, она сойдет за виток.
Аноним 21/11/15 Суб 22:55:25 #295 №231001 
>>230994
Ну так ведь в болванке спутник(-и). Он(-и) из неё вываливаются на орбите и пердят на свои места. Просто любопытна сама жизнеспособность концепции.
Аноним 21/11/15 Суб 23:06:12 #296 №231005 
>>231001
не путай болванку которую можно пнуть с ускорением в 300000g и которой похуй и спутник который нежно выводится на орбиту, не проходящую ни одной своей частью в нижних слоях атмосферы..
самое главное что ты упускаешь из виду - что болванка из пушки будет на орбите, которая вроходит собственно через саму пушку, и что более важно существенный(по воздействию) ее кусок находиться ниже 100 км.

но так ессно возможно, просто есть ряд проблем.
Аноним 21/11/15 Суб 23:31:07 #297 №231013 
>>230950
Значит их давление таки больше.
Аноним 21/11/15 Суб 23:47:03 #298 №231027 
>>231013
Попробуй обосновать этот вывод. Хуй знает, что ты там вкладываешь в понятие давления, например.
Аноним 22/11/15 Вск 05:05:38 #299 №231094 
>>231013
да больше, для того чтобы поток горючего мог перетекать в камеру. Но сила-мощность требуемая для перетекания горючего меньше (так как его самого по объему меньше), чем получается в процессе сгорания топлива, и истечении продуктов сгорания, так как продуктов сгорания при примерно томже (несколько меньшем) давлении, больше и значительно больше(раз эдак в несколько сотен), по объему, чем топлива.
Аноним 22/11/15 Вск 09:06:38 #300 №231104 
https://2ch.hk/zog/res/105939.html
черный рыцарь это правда?
Аноним 22/11/15 Вск 09:18:13 #301 №231107 
>>231094
Блядь. ЯННП.
Хули оно не попадает взад, ёба? Ведь при сгорании давление увеличивается.
Аноним 22/11/15 Вск 11:38:28 #302 №231122 
14481815084730.png
14481815084731.png
>>231094
>да больше
Ну чего "да"-то? Сам же дальше поясняешь, что наоборот, насос должен выдавливать топливо в камеру сгорания с чуть большим давлением. Остальное норм расписано.
Аноним 22/11/15 Вск 12:34:53 #303 №231129 
>>231104
Черный рыцарь - правда.
Вся лабуда, которую ему приписывают - нет.
Аноним 22/11/15 Вск 18:56:17 #304 №231209 
>>231107
У тебя что, двигатель из двух частей всего состоит, камеры и насоса, а снизу заварен намертво? Рабочее тело вылетает в СОПЛО, где давление меньше, а не обратно в турбину, где давление больше. Не знаю что еще можно здесь сказать.
Аноним 22/11/15 Вск 20:20:38 #305 №231234 
>>231122
>да больше
>Ну чего "да"-то?
косоглазие, показалось что этот >>231013 за топливо говорил
Аноним 22/11/15 Вск 22:53:28 #306 №231274 
>>231209
А как оно тогда летит? Может просто запилим оче быстрый насос, и будем строить гигантские водяные ракеты?
Аноним 23/11/15 Пнд 01:29:17 #307 №231317 
14482313578520.jpg
>>231274
норм норм, батарейки только не забудь
Аноним 23/11/15 Пнд 01:32:35 #308 №231318 
Уважаемые знатоки, вопрос: как называется звезда больше солнечной системы, если таковая есть?
Аноним 23/11/15 Пнд 02:38:01 #309 №231319 
14482354817900.jpg
>>231274
Можно и так.

Но если можно при этом использовать потанцевальную энергию химических связей жидкого энергоносителя, то почему бы так и не сделать. Будет гораздо эффектнее.
Аноним 23/11/15 Пнд 02:40:16 #310 №231320 
>>231318
Нет, таких пока не открыто, и вряд ли будет, ибо солнечная система довольно большая.

Даже сам Вояджер только-только вот на днях покинул её пределы.
Аноним 23/11/15 Пнд 08:14:28 #311 №231332 
14482556682570.png
>>231318
Самая большая по размерам нынче известная звезда радиусом дотянулась бы в нашей системе до орбиты Сатурна. Видеороликов и статей на тему самых больших звезд овердохуя, так что если тебе действительно интересно - вперед и с песней гуглить. Но маловероятно, что оно хоть сколько-нибудь поэтично. Что-то в духе FY Canis Majoris или типа того. Сугубо схематично, ибо слишком много звезд, чтобы каждой выдумывать оригинальное погоняло. Олсо настоящий спесачер уже давно не ведется на размеры звезд. Какой толк от размера, если плотность вещества на 99% этого объема в тысячи раз меньше плотности земной атмосферы. Таким макаром можно и на газопылевые облака дрочить. А вот звезды массой в 100+ солнечный уже куда интересней.
Аноним 23/11/15 Пнд 09:10:02 #312 №231334 
Что такое аккреционный диск у черной дыры, и почему он есть не у всех? От массы зависит или от чего-то еще?
Аноним 23/11/15 Пнд 10:08:59 #313 №231344 
>>231318
кажись квазар, но там может быть размером и с маленькую галактику
Аноним 23/11/15 Пнд 11:39:25 #314 №231352 
У меня такой вопрос: может ли человек обратить энтропию вспять, создавая разные архитектурные сооружения и прочие блага цивилизации? Или тут кроется какой-то подвох, и чтобы создать что-то с низкой энтропией придется вначале создать много энтропии и в результате энтропия всё равно возрастет?
Аноним 23/11/15 Пнд 11:48:31 #315 №231353 
>>231334
Аккреционный диск это диск вещества, которое падает по спирали на чёрную дыру. Есть не у всех, потому что не у всех черных дыр рядом есть вещество, которое могло бы ей притягиваться и падать на неё. Падает по спирали потому что изначально летит не в саму дыру, а рядом с ней пролетает (если например звезда пролетает мимо и часть вещества, то она изгибается под действием приливных сил и с неё отлетает к чёрной дыре, прямо как хвост у комет), и постепенно радиус орбиты вещества рядом с черной дырой уменьшается. Аккреционный диск в процессе падения разогревается и светится. Он может образоваться не только у чд, но и у нейтронных звезд, и даже в системах из двух обычных звезд.
Аноним 23/11/15 Пнд 11:52:00 #316 №231354 
>>231344
Ну если тебе "кажись" мелкобуква, то может не будешь свою пасть разевать?
Аноним 23/11/15 Пнд 13:14:47 #317 №231360 
Если поместить солнце в непроницаемый (даже для нейтрино) неразрушимый ящик, разогреется ли оно настолько, чтобы сгореть вплоть до железа и дальше?
Аноним 23/11/15 Пнд 19:45:02 #318 №231434 
>>231360
если его не будут покидать даже нейтрино, то солнце заполнит этот ящик, температура вырастит до неизвестного максимума, давление будет расти почти бесконечно, дальше сказать сложно, поведение материи в таких условиях не определены.
Аноним 23/11/15 Пнд 19:52:16 #319 №231439 
>>231434
>поведение материи в таких условиях не определены.
установиться динамическое равновесие между синтезом и распадом, что ты как маленький.
Аноним 23/11/15 Пнд 23:34:53 #320 №231542 
14483108935020.png
Гравитация это тоже самое что электромагнетизм? Прочел про эксперимент Фарадея, который доказал, что электромагнитное поле меняет траекторию движения фотонов. Получается, что на движение фотонов влияет и гравитация и магнитное поле одновременно?
Тогда почему считается, что на траекторию движения планет влияет только гравитация, а не их магнитное поле?
Аноним 23/11/15 Пнд 23:45:49 #321 №231548 
14483115495900.jpg
>>231542
https://ru.wikipedia.org/wiki/Электромагнитное_взаимодействие
>Электромагнитное взаимодействие заряженных частиц намного сильнее гравитационного, и единственная причина, по которой электромагнитное взаимодействие не проявляется с большой силой в космических масштабах — электрическая нейтральность материи, то есть наличие в каждой области Вселенной с высокой степенью точности равных количеств положительных и отрицательных зарядов.
Аноним 24/11/15 Втр 01:45:39 #322 №231570 
На викторине в универе попался вопрос про космос. Описания не помню, но ответ был "Бета Близнецов" или как-то так. И вот тут я понял, что кроме планет ничегошеньки не знаю о галактике. Как начать разбираться во всех этих бетах, альдебаранах и проч? Есть какой-нить научный фильм, может, годный?
Аноним 24/11/15 Втр 08:14:06 #323 №231595 
Почему саган выглядит как педофил?
Аноним 24/11/15 Втр 08:18:10 #324 №231596 
>>231570
Прочитай книжки Ричарда Фейнмана. Он хорошо поясняет за то, что знать названия чего-то нахуй не упало.
Аноним 24/11/15 Втр 08:24:41 #325 №231597 
>>231570
>Есть какой-нить научный фильм
Кек. Нет, братан. Хочешь сечь в астрономии - будь добр почитывать пыльные талмуды. Науч поп сейчас по черным дырам и прочим убийцам галактик угорает. Да и сама по себе астрономия в узком смысле слишком утилитарна, больше в вопросах навигации играет роль. Вот ответвления ее вроде планетологии, астрофизики и астробиологии дают простор для мечтаний. А с астрономией что, учи названия созвездий и звезд, их величины, склонения да прочие координаты. Как география, только без картинок и без гипотетической возможности побывать там, о чем читаешь.
Ну а если прям совсем интересно, но талмуды не привлекают, то попробуй вариант с цифровыми планетариями, их сейчас наплодили много. Stellarium, Celestia, Space engine - выбирай любой на вкус.
Аноним 24/11/15 Втр 09:18:24 #326 №231602 
>>231595
Почему бы тебе не получить нобелевскую премию за создания детектора педофила по внешности?
Аноним 24/11/15 Втр 11:13:47 #327 №231612 
>>228917
> Что мешает установить аткой РИТЭГ в смартфон
То, что добраться до кардиостимулятора надо делать операцию (ну или гарантированно подписаться (причём на пустом месте) на мокруху, если мы про всякую гопоту говорим). В то время когда батарейку из смартфона может любой дебил харкачер выковырять.

P.S. А плутоний, сука, токсичен.
Аноним 24/11/15 Втр 11:19:43 #328 №231614 
>>230985
Теоретически можно: http://physics.stackexchange.com/questions/35139/what-is-the-possibility-of-a-railgun-assisted-orbital-launch
Аноним 24/11/15 Втр 21:23:23 #329 №231692 
14483894033870.png
Аноним 24/11/15 Втр 22:26:43 #330 №231708 
>>231596
>>231597
Спасибо за советы, анончики
Аноним 25/11/15 Срд 01:37:31 #331 №231742 
>>229689
Бамп вопросу.
Аноним 25/11/15 Срд 01:45:53 #332 №231744 
>>231742
Слегка разминулись.
Аноним 25/11/15 Срд 02:54:26 #333 №231752 
>>231612
1. Цена.
2. Современные телефоны - говно, (в т.ч. и из-за цены), и живут пару лет. Ну и нахуя? Или разрабатывать стандарт и на новый телефон ставить старую батарейку? Дорого, кароч.
3. Аль-Кадырка покупает партию из тысячи телефонов, превращает батарейки в порошок, и неким образом во имя Аллаха и справедливости запускает его в систему водоснабжения. Получаем кучу неизлечимо больных людей, умирающих в мучениях. Контролировать батарейки невозможно, даже сейчас куча источников проёбывается, а потом всякие быдлоначальники из МЧС, не желая заёбываться с их утилизацией по процедуре, топят в ближайшем колодце, или бомж находит, и в лучшем случае сдаёт в металлолом. крипотная история про источник в стене панельки.тхт
Аноним 25/11/15 Срд 09:20:47 #334 №231781 
Бамп >>231352
Аноним 25/11/15 Срд 11:50:48 #335 №231793 
>>231352
>чтобы создать что-то с низкой энтропией придется вначале создать много энтропии и в результате энтропия всё равно возрастет
This
Няша 25/11/15 Срд 14:25:13 #336 №231831 
Тупае вопрос: Почему бы не делать гравиманёвр на орбите солнца? Мощность близкого подхожения гарантирует спокойный пролёт до края системы, если не въябется в астероид/
@сгоритвпизду.
Аноним 25/11/15 Срд 15:20:48 #337 №231840 
>>231831
Так ваш аппарат сгорит ведь.
Аноним 25/11/15 Срд 16:00:43 #338 №231845 
>>231840
А еще, даже если обуть аппарат в зеркальную броню, может случиться сорт оф эйрбрейкинг. У Солнца своя атмосфера. Може и не дохуя плотная, но дохуя протяженная.

Это если предположить, что аппарат проскочит где-то между протуберанцами и линиями магнитных полей так, что его не заденет.
Аноним 25/11/15 Срд 16:15:59 #339 №231846 
>>227164 (OP)
Посоны, я к вам из сайфача, предупреждаю сразу.

Где почитать, если есть, какие-такие минимальные аугментации стоит произвести человеку, чтобы нормас жить на комической станции? Чтобы удешевить калоницацию Марса?

Ибо терраформирование дорого и долго.

http://www.youtube.com/watch?v=xZr4nhY2Aj0

видео не рилейтед
Аноним 25/11/15 Срд 16:47:14 #340 №231849 
>>231831
> Почему бы не делать гравиманёвр на орбите солнца?
Потому что это половое излишество — ламинат на паркетный пол стелить: «…полет с Земли по гомановской траектории к точке поверхности Солнца, противоположной Земле, отстоящей от центра Солнца на расстоянии 0.00465 а.е., должен продолжаться 65.02 сут. Необходимая для этого скорость отлета с Земли 29.151 км/с есть минимальная скорость, обеспечивающая достижение Солнца.» (В. И. Левантовский. Механика космического полета в элементарном изложении)
Аноним 25/11/15 Срд 17:42:13 #341 №231879 
>>231831
Гравиманевр о Солнце невозможен с тела, находящегося на его орбите. Ты не понимаешь сути гравиманевра. Как можно использовать движение Солнца, если ты уже движешься вместе с ним, а, мразь?
Аноним 25/11/15 Срд 18:23:20 #342 №231886 
Школьник в треде. Не бейте, лучше обоссыте. Решил написать научно-исследовательскую по звездам. Теорию оформил, учитель сказал, нужны практические данные. Что мне делать? Связываться с местной обсерваторией просить у них консультации или в интернете можно найти что-то, что подойдет под задачу? Конкретно мне нужна двойная система с возможностью отследить ее и все это описать в работе.
Аноним 25/11/15 Срд 19:02:40 #343 №231900 
>>231886
Смоделируй в Universe Sandbox.
Аноним 25/11/15 Срд 19:47:16 #344 №231926 
>>231900
А если с ангельским все плохо?
Аноним 25/11/15 Срд 20:19:29 #345 №231940 
>>231926
Бери словарь. Потратишь больше времени, зато убьешь двух зайцев выучишь термины
Аноним 26/11/15 Чтв 00:40:49 #346 №232008 
Почему с земных телескопов нельзя сделать фотки Луны и увидеть там ровер, американский флаг и все остальное?
Аноним 26/11/15 Чтв 00:47:24 #347 №232014 
>>232008
>сделать фотки Луны и увидеть там ровер, американский флаг и все остальное?
Нахуя?
Аноним 26/11/15 Чтв 00:49:03 #348 №232017 
>>232014
Чтобы поставить точку в лунном заговоре хотя бы. Ну и что значит нахуя? Интересно же
Аноним 26/11/15 Чтв 00:58:22 #349 №232018 
>>232017
>лунном заговоре
Угу, взрослые дружащие с башкой дядьки будут выделять бабос на потакание маняфантазиям всяких придурков.

Ну а в целом, объекты такого размера в телескоп с Земли не различить.
Аноним 26/11/15 Чтв 01:00:10 #350 №232021 
>>232018
>Ну а в целом, объекты такого размера в телескоп с Земли не различить.
Даже глядя в охуенный хаббл? Луна же вот она рядом совсем
Аноним 26/11/15 Чтв 01:07:13 #351 №232023 
>>232021
>Даже глядя в охуенный хаббл? Луна же вот она рядом совсем
Будет детализация ~100 м/пиксель. Так что нет.
Аноним 26/11/15 Чтв 08:10:20 #352 №232072 
>>232021
Сука, не для разглядывания американких следов на луне сделан Хаббл, сколько раз можно поднимать эту тему, у него другие задачи, фокусные расстояния, аппаратура. Давайте будем в бинокль газету в руках читать.
Аноним 26/11/15 Чтв 11:39:55 #353 №232086 
Почему российские модули/корабли отделаны и засраны как плацкарт совкового поезда, а западные все такие белые-красивые-чистые? Прошу ответ без "потому что дикари".
Аноним 26/11/15 Чтв 11:44:09 #354 №232087 
>>232086
Даже материалы те же. Эта вот хуня из тексолита, с ламинированием в клеточку.
Аноним 26/11/15 Чтв 13:30:19 #355 №232094 
>>231926
Там технический язык. Думаю справишься.
Аноним 26/11/15 Чтв 18:17:14 #356 №232141 
>>231886
Для того, чтобы беспокоить занятых людей в обсерватории, тебе надо хотя бы сформулировать вопрос правильно. Что конкретно тебе нужно? Если у тебя чисто ньютоновская гравитационная система двух звезд (разделенная двойная), то либо смотреть каталоги и искать экспериментальные данные, что с хуевым английским это затянется, либо взять Universe Sandbox и назвать это "компьютерным моделированием", что вполне сгодится для твоих целей (если препод будет залупаться, скажи что полно научных работ с моделированием). А то знаешь, двойные всякие бывают - с пиздингом материала друг у друга, и прочими сложными процессами.
Аноним 26/11/15 Чтв 18:36:55 #357 №232144 
Спейсач, а зачем в русском отделении на мкс иконы висят? Я думал, что у космонавтов нет времени на такие вещи, как религия. И есть ли что-то подобное у американцев?
Аноним 26/11/15 Чтв 18:47:06 #358 №232151 
>>232144
Космонавты раза в два меньше загружены работой, чем астронавты. Вообще, большая часть космонавтов это просто хорошо подготовленное военное быдло.
https://youtu.be/x3ZwVjaMT38?t=3m44s
Аноним 26/11/15 Чтв 19:02:56 #359 №232161 
>хорошо подготовленное военное быдло
Ну так ведь это начальные требования, нет? Там определенное количество часов налетать на самолете, например.
>Космонавты раза в два меньше загружены работой, чем астронавты
Я правильно понял? Ты подразумеваешь, что русские работают меньше всех?
Но тот же Виноградов в одном из интервью говорил, что вообще ни у кого времени нет. Что ему какой-то сериал привезли 11-серийный и он его уже полгода смотрит
Аноним 26/11/15 Чтв 19:03:15 #360 №232162 
>>232161
>>232151
Аноним 26/11/15 Чтв 19:06:07 #361 №232166 
>>232151
И к чему ты видео кинул? Послушать их матерную речь?
Аноним 26/11/15 Чтв 19:13:22 #362 №232167 
>>232161
>>232166
зойчем ви когмите
Аноним 26/11/15 Чтв 19:13:39 #363 №232168 
>>232144
Давно не висят.
Аноним 26/11/15 Чтв 19:25:21 #364 №232172 
>>232168
К сожалению, висят. Как и крест.
Количество меняется постоянно. Пик миновал, но кто знает, что готовит будущее.
Аноним 26/11/15 Чтв 19:29:58 #365 №232173 
>>232161
http://novosti-kosmonavtiki.ru/forum/forum19/topic13873/
>Научная программа, к сожалению, вопрос больной. Этот вопрос наш экипаж даже поднял на традиционной беседе с руководителем Роскосмоса на Байконуре перед стартом. Например, еще полтора года назад медико-биологическая научная программа составляла 49 полетных экспериментов, а до полета дошло только 18. Остальные по различным причинам не состоялись, или отложились, или не успели.
>Доля, в количественном отношении, конечно меньше, чем хотелось бы. О научной ценности экспериментов, проводимых иностранными коллегами, еще можно и поспорить, но аппаратурная база, места для размещения приборов и научных комплексов у них организованы и продуманы гораздо лучше. Если на станции «Мир» у нас были специализированные научные модули и весь спектр научных направлений, то на МКС в этом плане гораздо хуже.
Аноним 26/11/15 Чтв 20:53:47 #366 №232204 
>>232141
Проблема уже решена. Спасибо за помощь.
Аноним 26/11/15 Чтв 21:37:15 #367 №232218 
Почему орбиты планет такие круглые? Я понимаю что они эллиптические, но почему эти эллипсы так похожи на круг? Почему нет планет, которые двигаются по очень узким эллипсам, то близко подходя к солнцу, то отлетая от него чёрт знает куда? Есть какой-то закон физики, который скруглял бы эллиптические орбиты через много времени?
Аноним 26/11/15 Чтв 21:42:05 #368 №232219 
14485633251150.jpg
>>232218
>Почему нет планет, которые двигаются по очень узким эллипсам, то близко подходя к солнцу, то отлетая от него чёрт знает куда?

https://ru.wikipedia.org/wiki/(90377)_Седна
>Седне необходимо примерно 11 400 лет для того, чтобы совершить полный оборот по своей сильно вытянутой орбите, которая в ближайшей от Солнца точке находится на расстоянии 76 а. е., а в дальней — на 900 а. е.
Аноним 26/11/15 Чтв 21:48:27 #369 №232222 
>>231831
Мы и так на орбите солнца, а оттормозиться к нему очень затратно, проще своим ходом улететь.

>>232008
Можно, но очень тяжело, нужны очень крутые телескопы, а они такой ерундой не занимаются.

>>232086
Потому что россия не очень, технологии в основном советские, надёжные, переделывать долго, дорого и опасно.

>>232144
У американцев руководители полётов тоже "благодарили бога" за то, что всё прошло хорошо с той миссией на луну, когда возникли неполадки и им пришлось возвращаться. Странные люди.
Аноним 26/11/15 Чтв 22:02:25 #370 №232226 
>>232218
>Почему орбиты планет такие круглые?
Во время формирования системы об них уебалось множество остаточных говен/пыли, что придало орбитам около-круговую форму. Ходят теории, что изначально планет было более чем дохуя, потом одни уебались в другие и получилось то что мы видим. Алсо, впоследствии случилась поздняя тяжелая бомбардировка, правда я не в курсе, имела ли она достаточную интенсивность, чтобы сильно изменить орбиту планет.

>Почему нет планет, которые двигаются по очень узким эллипсам, то близко подходя к солнцу, то отлетая от него чёрт знает куда?
Есть планетоиды с высокоэллиптическими орбитами, у которых афелий находится чуть ли не во внутреннем облаке Оорта. Есть также мнение, что на некоторые из параметров седноидов влияет некая здоровенная необнаруженная планета в одном из внешних поясов облака Оорта, через эффект Козаи.
Аноним 26/11/15 Чтв 22:24:07 #371 №232239 
Зачем на марсе нужен человек? Неужели есть задачи которые не под силу марсоходу? Вот чем занимались космонафты на луне? Нихуя не чем, просто показывали миру как они могут.
Аноним 26/11/15 Чтв 22:40:03 #372 №232245 
14485668037240.png
>>232222
Ты тоже благодаришь бога: спасибо = спаси бог. Странный ты человек.
Аноним 26/11/15 Чтв 22:44:10 #373 №232248 
>>231846
Отвечайте на мой ответ.

Если условия в космосе не идут у человеку, человек идет к условиям, приспосабливаясь.

Есть какие-то может институты или еще что-то?
Аноним 26/11/15 Чтв 22:46:16 #374 №232250 
>>232239
На марсе нужен марсианин, возможно с человеческими предками.
Аноним 26/11/15 Чтв 22:57:27 #375 №232256 
>>232245
Если звёзды больше земли, то как они могут падать на землю?
Аноним 27/11/15 Птн 00:27:56 #376 №232272 
14485732764840.png
Если к примеру ты попал за горизонт событий и тебя не распидорасило. Почему нельзя выехать от туда ведь караблю не надо развивать скорость чтобы вырваться из грав радиуса. Или я ошибаюсь.
Вот к примеру пикча. на первом шаге 1 км\с . на втором медленно падает до 0.5 км\с. на 3 увеличиваем до 1км\с. А на четвертом марс уже притягивает сильнее. Грубо конечно и топливо истратится пиздец как, но так же можно выехать с земли? И в чём пиздец у ЧД? Почему нельзя ехать от ЧД, а только к ней.

Прошу без всяких аналогий с канатами
Аноним 27/11/15 Птн 01:37:33 #377 №232297 
>>232272
>караблю не надо развивать скорость чтобы вырваться из грав радиуса
А что нужно сделать, помолиться богу?
Гравитационный радиус это и есть такое расстояние от центра массы тела, при котором тебе придется развить скорость света, чтобы вырваться.
Аноним 27/11/15 Птн 11:11:37 #378 №232368 
>>232272
Ну существование ЧД изначально вывели теоретически, по формуле. Существует формула определения скорости, которую нужно развить чтобы улететь с тела на бесконечность зная его массу и радиус (что-то вроде второй космической). И кто-то просто подставил в эту формулу скорость света и таким образом определил соотношение масса/радиус, которые нужны для такой ситуации, когда нужна скорость света, чтобы улететь с тела на бесконечность.
А вот что там с ускорениями я не знаю. Сомневаюсь что притяжение за горизонтом бесконечное.
Что ты хотел сказать пикчей я так и не понял.
Аноним 27/11/15 Птн 15:43:46 #379 №232411 
Часто вижу фото астронавтов на космических станциях, где у них куча ноутбуков приделанных к стенам, а что они на них делают?
Аноним 27/11/15 Птн 15:51:17 #380 №232414 
>>232368
> кто-то просто
Этим кем-то был маленький Карл Шварцшильд.
Аноним 27/11/15 Птн 15:51:55 #381 №232415 
>>232411
В свитер постят, двачуют капчу.
Аноним 27/11/15 Птн 18:03:07 #382 №232440 
>>232415
А я думал играют в доту
Аноним 27/11/15 Птн 18:04:52 #383 №232441 
14486366924040.png
>>232411
На линуксовых работают, на паре виндовых развлекаются: http://gizmodo.com/how-astronauts-use-laptops-on-the-international-space-s-1654962539
Аноним 27/11/15 Птн 19:15:32 #384 №232482 
>>232440
Может если только между себя, пинг великоват, канал неширок.
Аноним 27/11/15 Птн 21:20:01 #385 №232540 
Поясните за хуйню, нахуй надо они от Насы? https://www.youtube.com/user/blueoriginchannel/videos
Аноним 27/11/15 Птн 21:35:26 #386 №232543 
>>232540
Не понял вопроса. Если ты про Blue Origin - компания от основателя Амазон, лет 15 уже работает. Имеет хороший опыт в запиливании движков. Пилит туристический суборбитальный VTVL фаллос, РН и несколько двигателей.

Если ты про New Shepard - будут туристов катать наверх.
Аноним 27/11/15 Птн 21:46:08 #387 №232548 
>>227164 (OP)
Спейсаны, посоветуйте книг про эволюцию галактик и природу всяких астрономических объектов, желательно не жесткую научную литературу, но и не совсем для скучающих домохозяек. Из знаний в этой области есть только довольно поверхностные представления и курс школьной физики.
Аноним 27/11/15 Птн 21:50:33 #388 №232551 
>>232548
Зачем тебе скучные книги, когда в наше время всё доступно в виде наглядных и увлекательных видео? На постнауке ищи, и есть 2 сериала про космос от двух негров.
Аноним 27/11/15 Птн 22:08:03 #389 №232556 
>>232548
>довольно поверхностные представления и курс школьной физики
Смотри научпоп-программки. Книжки для тебя пока слишком круто.
Аноним 27/11/15 Птн 22:11:38 #390 №232558 
Вояджер ведь своей антеной в сторону земли смотрит? Почему на всяких артах и видюшках его ресуют летящим антеной вперёд?
Аноним 27/11/15 Птн 22:58:23 #391 №232574 
>>232558
Всем срать.
Аноним 28/11/15 Суб 00:52:36 #392 №232616 
>>232543
>>232540

они немного с улой которая мартин и локи которые немного с насой
Аноним 28/11/15 Суб 01:22:20 #393 №232627 
14486629404000.png
Оживим немного тред? Что самое ебовое в современной физике?

Все посмотрели Интерстеллар. Большинство тупых вопросов про то, как упасть на черную дыру. Видимо, "тренд" такой - ЧД в моде. Но есть некоторое говно (я его видел), которое настолько близко и реально, что ЧД так, хуита, уходят на второй план.

Подкину вам один феномен, только не обосритесь. И, в будущем, ускоряться не надо. А то вот та шел по раёну, чисто по-приколу, а когда РВАНУЛ С МЕСТА ЧАСТИЦЫ НАХУЙ ПОЛЕТЕЛИ, ФИЗИЧЕСКИЙ ВАКУУМ МАТЕРИАЛИЗОВАЛСЯ, ЗА СПИНОЙ ГОРИЗОНТ СОБЫТИЙ ВПИЗДУ ПОЯВИЛСЯ.

Да, любое ускоренное движение порождает некоторые веселости. Излучение Хокинга (теоретически) возникает над горизонтами ЧД, а это - возникает безотносительно горизонтов:
https://ru.wikipedia.org/wiki/Эффект_Унру
Аноним 28/11/15 Суб 01:40:10 #394 №232636 
>>232627
У меня бомбит от квантовой механики. Как, зная такую НЕХ, они до сих пор не стали волшебниками? А может стали, но скрыват от нас?
Аноним 28/11/15 Суб 07:29:31 #395 №232656 
>>232636
>У меня бомбит от квантовой механики. Как, зная такую НЕХ, они до сих пор не стали волшебниками? А может стали, но скрыват от нас?

Ничего: проживешь. Всем срать на твою "уникальность".
Аноним 28/11/15 Суб 09:17:20 #396 №232694 
Газовые гиганты полностью такие газовые, или под слоем газа есть какое-то твёрдое едро?
Аноним 28/11/15 Суб 09:56:56 #397 №232720 
>>232694
Может есть, а может и жидкое. Хрен его знает. Но ты спрашивай еще.

вряд ли газовое облако, размазанное по орбите, смогло бы стянуться в газовый гигант за время порядка формирования каменных планет, без каменной планетезимали же
Аноним 28/11/15 Суб 09:58:39 #398 №232722 
Как в космосе срут? Там же говно не будет падать в унитаз, отразится от стенок, пойдёт обратно, измажет весь анус, а потом будет летать по всему отсеку, воняя. Как с этим бороться? А если даже собирать его в пакетики и выкидывать, то это же будет несущаяся с 1 космической скоростью сраная куча космического мусора, притом не жалкий микрометеороид, а ебала, которая может пробить дыру в чём угодно на такой скорости, обосрать все искусственные спутники, и небо, и Аллаха. С этим же как-то борются? inb4: космонавты не срут.
Аноним 28/11/15 Суб 10:02:14 #399 №232724 
>>232694
https://ru.wikipedia.org/wiki/Металлический_водород
Аноним 28/11/15 Суб 10:03:30 #400 №232725 
>>232722
https://www.youtube.com/watch?v=EE1Y_IzOkgs
Аноним 28/11/15 Суб 22:53:34 #401 №232989 
14487404142760.jpg
Как так фотографируют? В каких участках земли такой вид?
Аноним 28/11/15 Суб 22:58:51 #402 №232991 
>>232720
>вряд ли газовое облако, размазанное по орбите, смогло бы стянуться в газовый гигант
Следствия из ТО допускают даже возникновение голых сингулярностей при коллапсе плотного газопылевого облака с большим угловым моментом. А ты говоришь "без каменюки хуй чё у вас выйдет".

Да, кстати, - то, что ядро Юпитера состоит из камня - не доминирующая гипотеза. Версия про шар металлического водорода ничем не хуже.
Аноним 28/11/15 Суб 23:33:18 #403 №233038 
>>232989
https://ru.wikipedia.org/wiki/Выдержка_(фото)
Аноним 29/11/15 Вск 00:37:07 #404 №233101 
>>233038
Ты хочешь сказать что невооружённым глазом такого не видно? Сомневаюсь.
Аноним 29/11/15 Вск 02:53:50 #405 №233243 
14487548304520.jpg
>>233101
>такого
Какого такого? Астрофотография бывает разной.
Аноним 29/11/15 Вск 04:29:11 #406 №233256 
>>233101
> такого не видно? Сомневаюсь.
Да хоть сомневай хоть не сомневайся, а в таких во фотачках - половина фаташопа
Аноним 29/11/15 Вск 11:59:52 #407 №233273 
>>232989
В южном полушарии, где больше звёзд и вдобавок вдали от ярко освещенного ночного неба крупных городов.
Аноним 29/11/15 Вск 12:58:40 #408 №233278 
>>233273
Достаточно второго условия, штатива и фотоаппарата с шириком.
Аноним 29/11/15 Вск 13:54:55 #409 №233290 
14487944955240.png
>>232297
>>232368
Я это написал к тому, что я не понимаю. Что если двигаться от центра масс меньше 2 космической. КК что столкнётся с небесной твердью? И почему такая хуйня не работает с ЧД?
Аноним 29/11/15 Вск 14:13:42 #410 №233295 
>>233290
Тебе не нужна вторая космическая, чтобы подпрыгнуть и оторваться от земли. Однако изначальная скорость будет "расходоваться" в процессе удаления, и если изначальная скорость была ниже второй космической, и её после начала полета никак дополнительно не увеличивали, то рано или поздно гравитация объекта притянет тебя к себе.
Аноним 29/11/15 Вск 15:58:20 #411 №233383 
>>233295
Но если её увеличить, то это же не значит что она станет 2 космической сразу. на пике например скорость может быть даже ниже 1.
Аноним 29/11/15 Вск 16:31:05 #412 №233388 
>>233383
Вторая космическая это та, которая в некотором смысле равна гравитации тела. То есть ещё чуть-чуть и ты никогда не вернешься на тело, а если скорость будет чуть ниже, тогда в дальней точке тело остановится и начнёт очень медленно разгоняться обратно.
На пике скорость в каждой точке разная (потому что в каждой точке на кк действует разная сила притяжения, ускоряющая/замедляющая кк в сторону массивного объекта), максимальная она в момент отрыва от земли, а минимальна в самой дальней точке от тела. Но это только в той ситуации, если мы стреляем из пушки. В реальности же полёты ракет сложней, скорость увеличивается и уменьшается постепенно за счет выброса части массы (топлива).
Аноним 29/11/15 Вск 19:06:51 #413 №233417 
>>233388
Так почему нельзя улететь от чёрной дыры? Как ты и сказал 2 космическая это скорость нужная снаряду пушки чтобы улететь с без атмосферного тела. Но почему нельзя дрейфовать с маленькой скоростью пока не вырвемся в точку где 2 космическая < скорости света?
Аноним 29/11/15 Вск 19:23:01 #414 №233421 
>>233101
Ты ебанутый на голову? Часто видишь длинные зеленые полоски в воздухе сами по себе?
Аноним 29/11/15 Вск 20:39:24 #415 №233428 
14488187645250.png
14488187645441.png
>>233278
Кстати, всегда интересовал этот вопрос. На фото, сделанных в более южных широтах, Млечный Путь пиздатее и выглядит немного по-другому, и вообще, еще пиздюком слышал, что Галактику лучше фотографировать в Южном полушарии.

Есть дохуя спецы?

На пиках - Исландия
Аноним 29/11/15 Вск 20:49:28 #416 №233430 
>>233417
>Но почему нельзя дрейфовать с маленькой скоростью
Ну блин, расписали же уже всё. Нельзя лететь со скоростью меньше световой, и улететь за горизонт. Твоя скорость будет постепенно уменьшаться (из-за притяжения), потом станет нулевой или отрицательной (ее вектор (сменит направление на противоположное - поменяется направление движения). Достигнуть же скорости света для тела, обладающего массой, невозможно.
Аноним 29/11/15 Вск 22:10:31 #417 №233476 
>>233417
Что значит "дрейфовать"? Космос не море, ты не сможешь просто поддерживать постоянную скорость, если не будешь ускоряться с силой, равной силе притяжения в данной конкретной точке. Сила притяжения массивного тела постоянно будет тормозить дрейфующий объект, поэтому дрейфовать ты долго не сможешь, в конце концов остановишься и начнешь падать обратно. А если речь идет о чд, тогда к тому моменту, когда 2 космическая будет меньше скорости света, у тебя уже не останется энергии, чтобы развить эту вторую космическую и вырваться от гравитации чд.
Основная сложность в гравитации на "поверхности" чд. Если ты сможешь её преодолеть, то и вырваться с неё сможешь, но в один шаг ты не улетишь, энергии не хватит (никакой массивный объект не может разогнаться до скорости света, насколько известно на данный момент, без выкрутасов в полями и прочего колдовства). Надо будет иметь как минимум ещё одного помощника неподалёку, который подхватит тебя и даст тебе дополнительную энергию, которая позволит вырваться из гравитации чд. Но это только в теории, при условии, что сила притяжения на "поверхности" чд не бесконечна и ты сможешь каким-то образом выдерживать и преодолеть её+добавку при ускорении в противоположном от неё направлении. И это очень топорный взгляд, который наверняка не учитывает ряд квантовых и релятивистских эффектов.
Очень возможно что я щас хуйню сказал
Аноним 29/11/15 Вск 22:25:18 #418 №233485 
http://www.youtube.com/watch?v=n-AdoTkXwbc
Аноним 29/11/15 Вск 22:26:28 #419 №233487 
>>233476
>Надо будет иметь как минимум ещё одного помощника неподалёку, который подхватит тебя и даст тебе дополнительную энергию
Ни что, попавшее под горизонт ("кисть руки товарища") оттуда уже не выйдет. Так что нет.

Излучение же Хокинга пока очень спорно, имеет ряд открытых проблем (от которых можно обосраться: такие криповые они), и без "квантовых <...> эффектов" там не обходится. Да и вряд ли ананасу понравится (через миллиарды лет) "вырваться" в виде слабенького свечения.
Аноним 29/11/15 Вск 22:56:23 #420 №233511 
>>233487
>Ни что, попавшее под горизонт оттуда уже не выйдет.
Часто можно встретить подобную фразу. Но вот почему? Понятно что в ни один корабль не обладает такими эффективными двигателями и таким запасом топлива, и люди не обладают технологиями, чтобы выжить хотя бы в окрестностях черной дыры под этим излучением и приливными силами. Но чисто гипотетически, я не вижу преград для обхода проблемы достижения второй космической, равной скорости света, двумя шагами.
Аноним 29/11/15 Вск 23:03:57 #421 №233514 
>>233430
А почему нельзя лететь от дыры? Вот у тебя есть ёба движок, с увеличением силы тяжести увеличивать и силу движка. Ок сила тяжести там ёбистая но и движок у тебя не слабый. направил сопло вниз. и так получилось что ты завис, приложил чуть больше силы уже полетел вверх, со скоростью куда меньше чем скорость света. Потом ты вылетелетел из под горизонта, и с той же скоростью 3 км\с прилетел к сраной земляшке. Зачем развивать скорость чтобы противостоять силе тяжести? Разве не достаточно Что бы она была просто положительной? Допустим там ёба УСП но ограничена же только скорость, а не ускорение. Еба звездолёт разгоняющейся до 0,1 с за 1 трилионную секунды, но развивающий лишь 0,05 с.
Аноним 30/11/15 Пнд 00:07:46 #422 №233529 
>>233511
>я не вижу преград для обхода проблемы достижения второй космической, равной скорости света
Ну хорошо, еще раз. Никакое тело, обладающее массой, не может достичь скорости света.

И втрое. В этом ответе содержится другой вопрос (несколько более сложный) и подсказка. Догадаешься, и сможешь ответить?
Аноним 30/11/15 Пнд 00:17:42 #423 №233532 
>>233529
>Никакое тело, обладающее массой, не может достичь скорости света.
Это понятно. Но в описанном мной решении телу не придется развивать скорость света. Точно так же как и космические корабли улетая с орбиты Земли никогда не летят со второй космической скоростью. Вторую космическую пришлось бы развивать только в ситуации, когда космическим кораблем является ядро, выпущенное из пушки, и тогда вторая космическая скорость была бы только в начальный момент полета.
Аноним 30/11/15 Пнд 01:44:51 #424 №233554 
>>233532
>Но в описанном мной решении телу не придется развивать скорость света
Область, ограниченная горизонтом ЧД - кусок, "вырезанный" из пространства-времени. По определению. Информация не может его покинуть. Ну и кроме того, ничто во Вселенной не может двигаться выше некоторой конечной скорости с (сейчас это скорость света). Последнее предложение - аксиома: без нее рушится каркас всего.

Энергия-масса в СТО тоже (как и скорость) связана преобразованиями Лоренца: Е = γmc2. Гамма-фактор, при световой скорости, не определен/бесконечен. Достичь бесконечной энергии мешают законы сохранения.

И да, твоё
>ограничена же только скорость, а не ускорение
не верно. Ускорение завязано на силу (например, через школьный Второй закон Ньютона). Чтобы ее прикладывать, нужна энергия. А достичь кинетической энергии выше той, которую можно развить на субсветовой скорости - нельзя из-за причин, описанных выше.
Аноним 30/11/15 Пнд 02:35:42 #425 №233558 
Возник тупой вопрос, но он мне не дает покоя. Если предположить, что наше солнце взорвется, а ударная волна движется в сторону Земли со скоростью, максимально приближенной к скорости света, то что увидят наблюдатели Солнца на Земле? Успеют ли они увидеть что либо перед смертью? Или просто резко умрут через 8 минут после взрыва, не успев ничего понять?
Аноним 30/11/15 Пнд 03:28:42 #426 №233592 
>>233558
Через16-ть: надо еще отраженному свету дойти.Скорость света конечна, а расстояние немалое. У себя-то, На Солнце, они быстро всё прочухают.
Аноним 30/11/15 Пнд 09:08:57 #427 №233632 
14488637374940.png
котаны, помогите узнать, что светило на небе

пик с краткой информацией прилепляю, может где-то косякнул с масштабами и так далее

время 7 00 утра Москва
Аноним 30/11/15 Пнд 10:03:10 #428 №233637 
>>233592
Куда твой отраженный свет идти должен, наблюдатели на Земле, про наблюдателей на Солнце никто не говорил.
Аноним 30/11/15 Пнд 14:00:30 #429 №233659 
14488812303910.png
>>233632
http://www.stellarium.org/ru/
Аноним 30/11/15 Пнд 15:54:34 #430 №233664 
>>233558
> Или просто резко умрут через 8 минут
Наблюдятлы на земляшке помрут сразу, резко и без возражений.
Потому что им никак, никогда и никаким образом не узнать, что произошло на солнце 8 минут назад.
Аноним 30/11/15 Пнд 17:47:31 #431 №233674 
>>233659
Спасибо,

странно Юпитер не выделялся так же сильно как эти две
Аноним 01/12/15 Втр 03:02:28 #432 №233978 
>>233664
Нет, это не так. Современная физика стоит на позициях близкодействия, и любое взаимодействие распространяется с конечной скоростью. Так что жди 8 минут, потом сдыхай.
[Космос и программирование] Аноним 01/12/15 Втр 15:22:25 #433 №234081 
>>227744
А погромисту, который мечтает о космосе, можно свою карьеру как-нибудь с этим связать?
Учусь и живу в ДС
Аноним 01/12/15 Втр 15:22:49 #434 №234082 
>>234081
>>227654
Аноним 01/12/15 Втр 16:24:41 #435 №234116 
>>234081
1. Уезжаешь.
2. Учишься.
3. Идешь работать в подрядчика NASA/девственницаголактеко/пейсикс.

NASA же на работу принимает только граждан.
Аноним 01/12/15 Втр 16:41:10 #436 №234124 
>>234116
Наса -- единственный путь?
А Роскосмос? Или частная Даурия? Там что, совсем все плохо?
Аноним 01/12/15 Втр 20:22:26 #437 №234153 
14489905462970.webm
Что на webm?
Аноним 01/12/15 Втр 20:51:32 #438 №234163 
>>234153
Металло-тентакли-пидоры-из-далёкого космоса ловят рактету на низкой орбите

Луна всё видит и сочувствует, но не может ничем помочь.
Аноним 01/12/15 Втр 20:51:46 #439 №234164 
>>234153
Выход полезной нагрузки из остатков ракеты? Камера, похоже, была на обтекателе.
Аноним 01/12/15 Втр 21:01:31 #440 №234167 
14489928919980.jpg
>>233592
>16-ть
16-стнадцать.
Аноним 01/12/15 Втр 21:21:26 #441 №234178 
Как на фотоаппарате olympus lens выдержку сделать?
Аноним 01/12/15 Втр 22:13:08 #442 №234210 
>>234178
Если у фотиков было такое сочетание кнопок, от которого какая-нить хуйня тут же ломалась/взрывалась я бы тебе именно ее и написал.
Но я таких фишек не знаю, поэтому живи
Аноним 01/12/15 Втр 22:15:37 #443 №234212 
>>234178
Инструкцию читай, пидор.
Аноним 01/12/15 Втр 22:27:57 #444 №234218 
14489980777490.png
>>227164 (OP)
Сап, спейсаны.
>Спрашиваем то, за что в других местах выдают путёвку в биореактор.
Очень на это рассчитываю.

У меня вопрос, суть такова:
Я немного балуюсь графоманией и у меня есть некая задумка, в упрощённом виду суть в том, что человечество узнаёт о грядущем лет через 200 пиздеце для планетки и напрягает все силы мира что бы успеть произвести средство спасения в виде мега-шипа, этакого нового ковчега, пригодного не только для того, что бы перевезти несколько миллиардов сосулек из точки А в точку Б, а по возможности сохранить и людей и животных в состоянии, при котором они смогут продолжать полноценно существовать и развиваться прямо в космосе, для этого должны быть всяческие системы обеспечения и прочее. По возможности в том или ином виде должны выжить и животные. Соответственно прошу
>анонимных ученых мирового уровня критически рассмотрели мою гениальную идею
и подсказали, как описать происходящее так, чтобы немного разбирающиесся в вопросе читатели (буде таковые найдутся) не сожгли в тот же момент сиё "произведение".
Каким может быть новый - космический - Ковчег?

1. Каким по размерам и дизайну должен быть кораблик? Мне представляется что-то типа самого большого старшипа с пика, но ещё крупнее. Кораблик только для межзвёздных перемещений на сильно досветовых скоростях, естественно, так что он, видимо, может быть любой формы, но что оптимальнее и проще? Да, таскать на него Земное наследие будут корабликами поменьше, уже способными находиться на околоземной орбите.
2. Как обосновать то, что создаётся один огромный корабль, а не пучок мелких (мне по сюжету удобнее так, иначе для создания подходящей атмосферы придётся устроить несколько аварий)?
3. Какие там должны быть основные системы, которым стоит уделить внимание, если закопаться в технику? Меня особенно интересует источник получения энергии (сколько её, кстати, понадобится на всё) в межзвёздном пространстве и принцип действия искусственной гравитации.

Ну и вообще - что имеет смыл упоминать ещё?
Аноним 01/12/15 Втр 22:37:05 #445 №234220 
14489986259280.png
14489986259301.png
>>234218
Не представляю как такую дырявую задумку можно воплотить во что-то убедительное. Разве что будет что-то совсем далёкое от реальности, уровня звёздных войн. Зачем ты вообще взялся за это дело, если ни черта в этом не понимаешь?
Аноним 01/12/15 Втр 22:39:29 #446 №234221 
>>234220
Это декорация. Она не так важна, как происходящее, но хочется, чтобы было правдоподобно.
Аноним 01/12/15 Втр 22:40:18 #447 №234222 
>>234218
Придумай другую причину для пиздеца.
Сейчас задетектить йоба-комету, которая может разъебошить Землю -- говно вопрос. Уничтожить ее за столько времени или отклонить с курса -- еще легче
Аноним 01/12/15 Втр 22:44:36 #448 №234224 
>>234222
А челябинский метеорит?
В любом случае я не говорил ничего о комете. Тем более, что колонизировать Марс и Венеру при таком раскладе было бы в сто раз проще, чем лететь КЕМ. Я предполагал проблемы с Солнцем, возможно даже по вине человека, но это пока под огромным вопросом.
Аноним 01/12/15 Втр 22:56:17 #449 №234225 
>>234224
>А челябинский метеорит?
Так он не разъебал же даже ничего, он сгорел на 99.9%
>В любом случае я не говорил ничего о комете.
Сорри, это я почему-то сразу о кометах подумал (раздел же космический).

Хотя, в принципе, можно любое допущение сделать. Уир же тоже допустил шторм на Марсе (правда, там выбора особо не было, как устроить пиздец).

Можешь почитать "Древний" Тармашева (достаточно первую часть). Там чуваков спасли чем-то вроде анабиоза. Алсо, мне оче нравится, как автор в будущем подошел к вопросу вооружения людей. Там нужно было всячески беречь жизнь в бункерах и поэтому огнестрел полностью заменили на парализаторы
Аноним 02/12/15 Срд 00:24:48 #450 №234263 
>>234081
насилуй сопромат, вычметоды и физмат
хз это я в рамках гипотезы, без док
Аноним 02/12/15 Срд 00:27:24 #451 №234264 
Челябинский метеорит разъебал бы очень много, если бы взорвался над, собственно, Челябинском.

500 КТ ТНТ на высоте 30 км. Если посчитать исхохя из квадратичного убывания энергии с дистанцией, ты было бы похоже на 14 КТ на высоте 500 м -- как раз столько, сколько было в Хиросиме (580 м, 15 КТ).

Вкратце -- был бы полный пиздец.
Аноним 02/12/15 Срд 00:45:19 #452 №234274 
>>234218
>что человечество узнаёт о грядущем лет через 200 пиздеце для планетки и напрягает все силы мира что бы успеть произвести средство спасения в виде мега-шипа, этакого нового ковчега,
извини хуйня

>Каким может быть новый - космический - Ковчег?
хабитат https://en.wikipedia.org/wiki/O%27Neill_cylinder
найди источник, там неплохо расписано как за техническую часть, так и за принципиальные вопросы , достаточно по верхам, но для костяка тебе хватит, двести лет умнож там цифры на 10 будет похоже

>межзвёздных перемещений на сильно досветовых скоростях
>принцип действия искусственной гравитации.
водка без пива, деньги на ветер - понимаю.

>сколько её, кстати, понадобится на всё
бери сотка киловатт на человека, не промахнешся

тонн 50 на чела по массе в среднем шип, будешь динозавров и прочую фауну накинь тоже им.

подумай также что это за пиздец для планеты, который вынуждает покинуть солнечную систему, нахуа так сказать. http://www.youtube.com/watch?v=W4UfmOHjiJg

>Мне представляется что-то типа самого большого старшипа с пика, но ещё крупнее.
если им приходится напрягать всю жопу, то имеет смысл вернуться к кораблям шарам - как оптимальным формам в соотношении объем, площадь поверхности (и соотвественно материалов и работы)

но вообщем можешь не запариваться, или тебе еще придется продумать гораздо гораздо больше чем ты даже думал спросить.
Аноним 02/12/15 Срд 00:55:39 #453 №234286 
>>234225
>А челябинский метеорит?
>Так он не разъебал же даже ничего, он сгорел на 99.9%
траектория входа была неудачная, как бы цынично это не звучало.

>>234224
>что колонизировать Марс и Венеру при таком раскладе было бы в сто раз проще, чем лететь
короче не заморачивайся, просто дай вводную как дано: корабль, превозмогание, намзывание лыж из системы и дальше уже расписывай пролюбовь, если это просто антураж.

>>234264
чето я сегодня слоупок, все все уже написали
Аноним 02/12/15 Срд 00:58:10 #454 №234287 
>>234264
Но разве под взрывом случилось то, что случилось в хиросиме?
Аноним 02/12/15 Срд 01:01:41 #455 №234291 
>>234225
>Так он не разъебал же даже ничего
Тем не менее его не заметили. Не думаю, что заметят и что-то принципиально более крупное.
>раздел же космический
Любая катастрофа планетарного масштаба имеет отношение к космосу. Конкретно на Солнце натолкнул рассказ Дмитрия Градинара "Первый день весны" с описание леденящего душу безпросветного пиздеца. Ну и ещё - "Вспышка" Желязны, хотя там всё не так страшно и выкладки некоторых учёных (выступающих время от времени в прессе и на ТВ, в качестве приглашённых консультантов), говорящих, что влияние солнца и природных процессов (болотные испарения, вулканы, etc) на климат вообще и парниковый эффект в частности намного сильнее любых воздействий со стороны человека, хотя у меня слишком мало знаний в этой области, что бы оценивать их истинность.
>можно любое допущение сделать.
Можно. Но если я заявлю, что это был монументальный корабль, выпиливавшийся сто лет напильником из железа, добытого с пролетавших мимо асетройдов, сферической формы и имел 100км в диаметре логика говорит, что сфера почти оптимальна, хотя я бы хотел что-нибудь такое, от чего космофаны просто от описания кончали бы, а шарик это таки скучно, а мне спейсаче-маньяк заявят, что он ссыт на меня и произведение говно потому, что на корбле такого объёма за вычетом всех систем поместится не 100 млрд живых существ, а только 10 и то, если животные останутся сосульками, то будет неприятно. Вроде и по тексту претензий нет, но декорации - косячные. В любом случае мне не сложно провести тут какое-то время для того, что бы создать предельно верное окружение. и я надеюсь, что "анонимные ученые мирового уровня", упомянутые в ОП-посте потратят несколько минут своей драгоценной жизни на такую теоретическую задачку. При этом я хочу получить ощущение сверх-задачи, практически непосильной для человечества. Такой, что изменит сознание надрывом всех сил. И такой... Впрочем, дальнейшее окажется спойлером, а давать спойлер того, что существует в вид пары черновиков, полагаю, не стоит.
Но такое пространство для идей и вопросов, что я даже не представляю объём итогового произведения. Как изменится жизнь людей? Что произойдёт из-за столь близкого соседства культур? Будут ли люди воевать? Что будет означать власть? Скоро ли все забудут об опасностях путешествия и смирятся? Продолжится ли прогресс и как он будет двигаться? А что станет с бизнесом и экономикой? И какова окажется итоговая цель, а главное - что ощутят те, кто спустя многие поколения её достигнет? И главный вопрос я тут не перечислил, эти - относительно бытовые.
Полагаю, это будет несколько рассказов с суммарным объёмом на небольшую повесть. Если очень повезёт, то будет что-то вроде "Города" Саймака.
>Уир же тоже допустил шторм на Марсе
Не читал, к сожалению.
>Можешь почитать "Древний" Тармашева (достаточно первую часть).
Спасибо, почитаю. Хотя я думал, что если что-то такое и было, то самое реалистичное надо внимательно искать у Лема, которого я прочитал меньше половины.

Кое на что меня на самом деле вдохновили "Wall-E" и рассказ Уильяма Тенна "Жили люди на Бикини, жили люди на Атту", но слишком много осталось вопросов после ознакомления, там это всё берётся очень поверхностно (интересные фразы о масштабах есть у Френка Герберта, но он там сильно о своём).
К слову, я не думаю, что коридоры и каюты - нормальное решение. Должно быть много открытых пространств, а животные в идеале должны жить в условиях, очень близких к естественным, пусть и на ограниченных локациях (как в Голодных Играх грубо говоря). Ещё нужно что-то придумать с перемещением, а то иногда это смотрится крайне упорото. Например, я помню «Космический госпиталь» Джеймса Уайта - не знаю, как тогда, а сейчас я прихожу в недоумение от того, что за цирк с конями они там устраивали при простом перемещении по уровням (конечно, мне такое не грозит, но накосячить можно и тут).
Аноним 02/12/15 Срд 01:04:00 #456 №234292 
Почему скафандры для выхода вкосмос такие толстые? Что там насовано, защита от радиации?
Аноним 02/12/15 Срд 01:06:55 #457 №234294 
Вот этот чувачок хорошо поясняет за метеориты, рассказывает про размеры для достижения нужных эффектов.
https://youtu.be/Hp9WrhjGC20
Аноним 02/12/15 Срд 01:11:12 #458 №234300 
>>234292
Там много чего насовано. Очень технологичная штука. Было видео где рассказывается о каждом слое, из чего сделан и для чего нужен, но хуй знает как его найти.
https://youtu.be/VsdoJy8rzZg
Аноним 02/12/15 Срд 01:16:38 #459 №234305 
>>234264
>Если посчитать исхохя из квадратичного убывания энергии с дистанцией
Ударная волна ослабевает по кубу дистанции.

Ничего не будет.
http://nuclearsecrecy.com/nukemap/?&kt=500&lat=55.1732694&lng=61.4103841&hob_opt=2&hob_psi=5&hob_ft=98425&fireball=0&rem=&zm=11
Аноним 02/12/15 Срд 01:18:48 #460 №234306 
>>234291
Зачем тебе животные на корабле? Их можно воссоздать генной инженерией, достаточно просто записать их последовательность днк. А перевозить больно муторно.
Аноним 02/12/15 Срд 01:21:29 #461 №234308 
>>234291
>монументальный корабль, выпиливавшийся сто лет напильником из железа, добытого с пролетавших мимо асетройдов, сферической формы и имел 100км в диаметре
А мне вот, наоборот, сфера не нравится. Я бы кончил, если бы это было что-то наподобие мкс, но в 100 раз больше. С жилыми модулями, столовыми, всякими учебными классами, импровизированными фермами и чтобы можно было грабить корованы.

Можно еще добавить драматизма, типа, когда людей туда засылали на Союзах, не все долетали из-за хуевого здоровья/невыдержавшего сердца
Аноним 02/12/15 Срд 01:28:38 #462 №234314 
>>234300
Инструменты слишком железные, могли бы сделать что-нибудь модульное и легче, раз каждый килограм стоит как квартира в Нью-Йорке.
Аноним 02/12/15 Срд 01:30:05 #463 №234315 
>>234291
>При этом я хочу получить ощущение сверх-задачи, практически непосильной для человечества. Такой, что изменит сознание надрывом всех сил.
это косяк, это не выпиливание напильником 100км, выпиливание на это не тянет. Самое надрывное преодоления человека, это он сам. Тут вообщем думать надо.

а человека если живым бери гето 10 тыщ кубометров - норм в принципе, если тушками то сам знаешь два на один

>сфера почти оптимальна
норм это ретро а ретро возвращается, постоянно.

>И главный вопрос
я так понимаю не вознесутся ли они или превратятся в хивмайнд. посмотри старгейт чтоли, атлантис.

>Можешь почитать "Древний" Тармашева (достаточно первую часть).
>Спасибо, почитаю. Хотя я думал, что если что-то такое и было, то самое реалистичное
это немного в разные стороны, у Тармашова превозмогание во все поля, какраз по теме того что ты там про жизнь писал.

>К слову, я не думаю, что коридоры и каюты - нормальное решение. Должно быть много открытых пространств, а животные в идеале должны жить в условиях, очень близких к естественным, пусть и на ограниченных локациях
да да того хабитат смотри - это самое годное из того что ты хочешь

также самый тупой и не заданный вопрос, не смотря на то что мне приятно нести свет истины и все такое. На этом странном форуме есть также рзадел научной фантастики, sf , можешь там топик сделать, может взлететь.
также там еще не утонуо пара топиков - один про космические войны, я бы на твоем месте почитал его.
Аноним 02/12/15 Срд 01:33:23 #464 №234318 
>>234305
с фигали по кубу, да фронт конечно размазывается, но не настолько же.

>>234306
>достаточно просто записать их последовательность днк
не достаточно
>А перевозить больно муторно.
зависит от условий, не обязательно в виде зоопарка.
Аноним 02/12/15 Срд 01:36:34 #465 №234320 
>>234308
на тему евы онлайн ничего не ... хм пойдука я в сайфаич спрошу, надо подпитать свои скрепы
Аноним 02/12/15 Срд 02:45:31 #466 №234365 
>>234318
Две степени на расширения фронта по сфере, одна - на затухание ударной волны в атмосфере. Световое излучение в ясную погоду ослабевает по квадрату расстояния. У зарядов >1 МГт, зарядов радиус поражение световым изучением обгоняет все остальные, и чем больше заряд тем больше обгоняет.
Аноним 02/12/15 Срд 03:41:52 #467 №234375 
>>234225
>Так он не разъебал же даже ничего
Важнее, что его не заметили. И если бы он был в несколько раз крупнее - всё равно не заметили бы скорее всего, а всякие лютые хорроры травят про то, что к моменту касания имеет буквально десяток-другой метров в диаметре.

>>234274
>извини хуйня
В чём именно?
>там неплохо расписано как за техническую часть, так и за принципиальные вопросы, достаточно по верхам
Спасибо! И, кстати, на ру-вики статья не хуже.
>двести лет умнож там цифры на 10 будет похоже
В смысле 2 000 лет? Я думаю, 5-6 поколений с лихвой хватит на любую сверх-задачу, если не распылять ресурсы.
>водка без пива, деньги на ветер - понимаю.
Какие-то не очень очевидные метафоры. Сверхсветовые скорости, гиперпрыжки и прочее мне кажутся слишком большой условностью, поэтому скорости - реалистичные, а гравитация - для того, чтобы можно было продолжать жить в нормальном режиме. Отчего-то вспомнился "День рождения кота" Леонида Кудрявцева, там раскрывается темя перелётов на досветовых скоростях.
>бери сотка киловатт на человека, не промахнешся
Это со всеми мощностями - жизнеобеспечение, восстановление кислорода, обогрев и т.п.? И главное - ведь потребуется почти что вечный двигатель, а это кагбе косяк.
>пиздец для планеты
Скорее всего проблемы с Солнцем, в этом я почти уверен, иначе зачем сваливать слишком далеко?.. За видео спасибо - гляну.
>имеет смысл вернуться к кораблям шарам
Так и полагал в принципе. Хотя эти цилиндры смотрятся тоже интересно. Напомнили "Элизиум: Рай не на Земле", только приспособленные к дальним перелётам про Стэнфордский тор прочитал после того, как это написал.

>но вообщем можешь не запариваться, или тебе еще придется продумать гораздо гораздо больше чем ты даже думал спросить.
Для меня это хорошо. Я куда больше люблю научную фантастику, чем беллетристику без смысла (у Джеймса Уайта было кое-что интересное помимо техники, хотя уровень, конечно, весьма бульварный) и если получится добавить правдоподобности - будет просто супер.

>>234286
>короче не заморачивайся, просто дай вводную как дано
Я не хочу заморачиваться, но я техногик и не могу себе позволить гнать полную ересь. В идеале я хочу получить такой сюжет, который при определённых условиях и с рядом оговорок может произойти на самом деле.
>расписывай пролюбовь
Без пролюбовь, наверное, не обойдётся, она будет обязательно и даже очень, но только если в одном единственном сюжете - о сломе сознания первых поколений. Не знаю... Клиффорда Саймака читал, например? Написал очень много - гуманист, все дела, но как часто он писал о любви? Я так навскидку среди многих десятков его рассказов и нескольких романов помню только про "подгрести с непристойным предложением" в СВЕРХОХУЕННЕЙШЕМ "Почти как люди".

>>234306
>Зачем тебе животные на корабле?
Чтобы в достаточной мере сохранить разнообразие жизненных форм, а поскольку люди будут существовать в таком положении неопределённо долго - и животных следует иметь вживую. Ну и просто из гуманистических соображений - оставлять их на скорую, гарантированную смерть не захочется. Потом
>Их можно воссоздать генной инженерией, достаточно просто записать их последовательность днк.
На сегодня не получилось пока, а во время подготовки все будут слишком заняты другим - это раз. И два: если ты возьмёшь образцы у десятка особей, то они вымрут за несколько поколений от кровосмешений, нужно же что-то более радикальное. И вывезти их всех - достаточно радикально.

>>234308
>мне вот, наоборот, сфера не нравится
Мне тоже, но как справедливо заметили выше:>>234274
>если им приходится напрягать всю жопу, то имеет смысл вернуться к кораблям шарам - как оптимальным формам в соотношении объем, площадь поверхности (и соотвественно материалов и работы)

>Я бы кончил, если бы это было что-то наподобие мкс, но в 100 раз больше.
Слишком сложно. Когда задача получить оптимальный результат на такое не пойдёт никто.
>С жилыми модулями, столовыми, всякими учебными классами, импровизированными фермами
И не только. С относительно открытыми крупными залами, урезанными, но существующими прериями и так далее. Очень грубо: Корускант на реактивной тяге и не только с гуманоидами на борту.

>>234315
>Самое надрывное преодоления человека, это он сам. Тут вообщем думать надо.
Примерно в этом и суть. Но что-то должно произойти, что бы изменился не человек, но человечество? Я по правде говоря помню достаточно радикальный сценарий только в "Хранителях" - как в фильме, так и в оригинале (оригинал лучше). Вроде, ещё есть "День, когда Земля остановилась", но у меня всё не доходят до него руки и он влияет не на всех (а возьму-ка я и пойду его сейчас смотреть - того, который классический - 51-го года). Ведь смысл в том, что такое перемещение коснётся каждого отдельного индивида в том числе наряду с учёными из Европы это будет и какая-нибудь рандомная алкашня, и всяческие радикалы от Даиш до ИРА, но будут и туземцы, дикари из африканских племён, из Австралии и т.д. А ещё культура (упоминал вскользь) - ты, конечно, можешь собрать экспонаты многих музеев, но ведь не всех? Как ни старайся. И тем более ты не сможешь взять с собой Пирамиды и всякие статуи и храмы Будды. ЮНЕСКО придётся заняться реальной работой.
>если тушками то сам знаешь два на один
Тушками не интересно. Интересно - когда общество продолжает жить, но по новому.
Например, подумай сам для затравки: все видели, во что превращают пространство вокруг себя мигранты в Европе, прекрасное доказательство того, что сознание определяет бытие. Но как они же будут жить в соседстве с иными людьми в очень близких условиях? Скучкуются и образуют варварское гетто? А что будет со всякими радикалами? В общем есть о чём подумать.
10 000 м^3 на человека и раз в 10-100 больше на животное в диких зонах, можно с имитацией неба довольно невысоко. Как быть с птицами - представляю весьма примерно.
>я так понимаю не вознесутся ли они или превратятся в хивмайнд. посмотри старгейт чтоли, атлантис.
Посмотрю, хотя другие старгейтсы я видел (сериалы не целиком, на ТВ3), там ничего особого не помню (сериалы и вовсе показались унылыми, а фильм - ничего так). На самом деле с учётом того, что маловероятно, что кто-то из присутствующих когда-то наткнётся на моё произведение (и даже нет 100% гарантии, что я его добью успешно), пожалуй, можно озвучить (только имейте в виду, что это не просто замазанный текст, а натуральный спойлер): а, собственно, когда они доберутся до некой системы, в которой разведчики обнаружат подходящую планету, они спустятся на неё и посмотрят в небо, планета будет очень похожей на описания Земли, оставленное далёкими уже предками, почти не заселена и можно будет выпускать животных и людей, не боясь нарушить экологического баланса, и вот в этот момент люди должны будут сделать очень важный для себя выбор - как дальше жить? Расселиться по планете и восстанавливать культуру, или просто вернуться на корабль и продолжить бороздить космические просторы? Не знаю, смогу ли я сам ответить на такой вопрос...
>у Тармашова превозмогание во все поля, какраз по теме того что ты там про жизнь писал.
Превозмогание это не самоцель, я недостаточно разделил, видимо. Само путешествие будет относительно нормальным (насколько может быть таковым), но за 200 лет Великого Строительства сознание должно будет измениться, вот эти 200 лет и будут концентрированным сверх-рывком, который не может пройти без последствий. Конечно, события столь давние, как их не документируй для потомков сложатся в миф, ты не сможешь почувствовать той эпохи, как бы кто-то не старался передать её дух и это Строительство напряжёт всех и поднимет волну, но что станет с людьми потом, когда будет дан старт, люди увидят начало конца, поймут, что сами они чудом, но выжили и волна схлынет, очевидно она что-то оголит при этом. Об этом и будут остальные рассказы цикла.
По крайней мере я так представляю. Все помнят, что, например, получилось у Желязны в "Хрониках Амбера" и все интересующиеся - прекрасно знают, что именно задумывалось. Между ними пропасть.
Аноним 02/12/15 Срд 05:43:28 #468 №234379 
Кстати, о сфере/цилиндре. Мне тут подумалось, что сферу не получится сделать концентрической? Я сначала думал о чём-то многоуровневом, завёрнутом в трубочку/шарик, но тут понял, что если раскручивать это для создания гравитации, то ближе к центру гравитация будет во много раз меньше, чем у обшивки. И у сферы речь пойдёт даже не о центре, а об оси, значит гравитация с нужным ускорением будет доступна только на небольшом слое. У цилиндров с этим в принципе получше. Но всё-таки есть ли хоть какая-то возможность добиться появления чего-то похожего на гравитацию без вращения? Хотя бы сильно теоретически, но без торсионных полей и совсем уж теоретической части теории струн.
Аноним 02/12/15 Срд 07:28:00 #469 №234384 
>>234292
Отражающий слой, два механических слоя на прорыв (основной и резервный), два механических слоя для удержания давления, слой стяжек для удержания формы под давлением, технический слой с проводами и прочей ерундой, два нижних слоя для контакта с телом, из них один с водяными трубками для контроля температуры. Это как минимум. А вообще с виду толстые потому, что раздувает изнутри. Там есть стяжки чтобы форму держало, но от раздувания не спасает.
Аноним 02/12/15 Срд 07:31:08 #470 №234385 
>>234153
Похоже на суборбитальный пуск.
Аноним 02/12/15 Срд 08:08:12 #471 №234389 
14490328925060.jpg
Что произойдет с солнцем, если все его излучение экранировать во внутрь и не давать выходить наружу?
Аноним 02/12/15 Срд 12:57:47 #472 №234434 
>>234389 - >>231360
Аноним 02/12/15 Срд 15:43:18 #473 №234603 
>>234291
>безпросветного
Ты бы, графоман, не обсирался бы так.
Аноним 02/12/15 Срд 15:53:47 #474 №234623 
Можете объяснить, про цыкл жизни звезды. Особенно интересует последние стадии которые там переходные с белого карлика в нейтронного, хочется знать именно каким образом они так переходят. Как формируются черные дыры со звезд и т.п. На какой стадии находится наше Солнце сейчас (именно с точки зрения терминологии).
Аноним 02/12/15 Срд 16:06:20 #475 №234630 
>>234623
https://ru.wikipedia.org/wiki/Звёздная_эволюция
https://ru.wikipedia.org/wiki/Диаграмма_Герцшпрунга_—_Рассела
Аноним 02/12/15 Срд 20:00:01 #476 №234810 
>>234153
http://www.popmech.ru/technologies/232462-video-kak-otdelyaetsya-stupen-rakety-v-kosmose-unikalnaya-semka/?utm_source=popmech&utm_medium=rss&utm_campaign=public-all-articles
Аноним 02/12/15 Срд 23:47:06 #477 №234870 
Что будет, если столкнуть между собой несколько железных шаров - остатков звёзд? Будет ли взрыв сверхновой, а затем образование черной дыры?
Аноним 03/12/15 Чтв 00:14:49 #478 №234882 
>>234870
>железных шаров - остатков звёзд
Пиздец.
Аноним 03/12/15 Чтв 01:37:34 #479 №234893 
14490958548760.jpg
>>234870
>столкнуть между собой несколько железных шаров
Некоторые сломаются, остальные проебутся.
Аноним 03/12/15 Чтв 02:18:51 #480 №234899 
>>234870
> Будет ли взрыв сверхновой, а затем образование черной дыры?
Образование ЧД произоидет, если суммарная масса превзойдёт предел оппенгеймера-волкова. но если скорость столкновения будки слишко большая, на ЧД может не хватить - железо расплескается по врему вакууму

Взрыва свехновой - не-бу-дет, тердо и чотко.
Хотя взрывное энерговыделение - возможно, опять таки, все дело в относительной скорости шаров
Аноним 03/12/15 Чтв 09:58:07 #481 №234916 
14491258871530.png
Может ли существовать такая система, что бы была планета на которую палят 1 двойная и 1 Голубая звёзды, так что темнота на планете наступала когда звёзды закрывал бы газовый гигант на внутренней орбите?
Аноним 03/12/15 Чтв 11:03:47 #482 №234919 
>>234899
Шары будут очень мягко присоединяться друг к другу, как капельки,сливающиеся, вращаясь друг вокруг друга.
>Взрыва свехновой - не-бу-дет, тердо и чотко.
По-че-му? Должен же быть. В какой-то момент начнётся резкое сжатие под действием гравитации, атомы не смогут сопротивляться ядерным силам, радиус железного шара начнет стремительно падать, а затем сжатие резко остановится, и если массы будет недостаточно для ЧД, то пойдет взрывная волна, как если резиновые шарики столкнулись о стену. Это и будет взрывом сверхновой. По крайней мере я так это себе представляю.
Аноним 03/12/15 Чтв 11:42:42 #483 №234923 
>>234919
>под действием гравитации, атомы не смогут сопротивляться ядерным силам
Каким силам? Это два разных взаимодействия, вообще-то.
Аноним 03/12/15 Чтв 11:53:42 #484 №234926 
Черные дыры существуют? Просто где-то читал, что официально ни одной не найдено, и что известные чд могут быть просто похожими объектами. Тобишь, они идеально ложатся в теорию, вот про них и стали говорить.
Аноним 03/12/15 Чтв 12:01:00 #485 №234928 
>>234916
Я ничего не понял, рисуй картинку.
Аноним 03/12/15 Чтв 12:40:48 #486 №234931 
>>234926
Ты все верно читал, есть объекты-кандидаты, но как тут однажды отвечали "если объект плавает как утка, выглядит как утка и крякает как утка, то не будет антинаучным принимать его за утку". С чд точно также. Есть объекты, которые не излучают свет, которые заметно линзируют объекты на линии наблюдения и т. д.
Аноним 03/12/15 Чтв 12:49:05 #487 №234932 
>>234931
>Есть объекты, которые не излучают свет
Все кандидаты в ЧД, насколько знаю, скрыты за мощными аккреционными дисками, и светят мощнее любой звезды. В коротких волнах, во всяком случае.
Аноним 03/12/15 Чтв 16:29:27 #488 №234950 
14491493675630.png
>>234928
Аноним 03/12/15 Чтв 16:50:39 #489 №234955 
>>234950
Чтобы тень от гг закрывала планету и при этом у планеты была собственная орбита? Нет пути!
Аноним 03/12/15 Чтв 17:43:51 #490 №234973 
14491538318720.webm
>>234926
Формально ты хуй проверишь что это, так как они по определению ничего не излучают, мы видим только аккреационные диски. Так-то с большой уверенностью их уже обнаруживают и давно следят с попкорном за экшеном в центре нашей галактики.
Аноним 03/12/15 Чтв 18:52:02 #491 №234988 
>>234973
>>Просто где-то читал, что официально ни одной не найдено, и что известные чд могут быть просто похожими объектами. Тобишь, они идеально ложатся в теорию, вот про них и стали говорить.
>Формально ты хуй проверишь что это, так как они по определению ничего не излучают, мы видим только аккреационные диски.
Формально, в теории нет звезд с массой в 4 млн. М. Так что по гравитационному влиянию кое-что можно вывести.
Аноним 03/12/15 Чтв 23:07:33 #492 №235137 
>>234434
нихуясебе, ты не ленивый
Аноним 03/12/15 Чтв 23:27:44 #493 №235141 
есть что почитать на счет растений и рыбок на мире или мкс.
Аноним 04/12/15 Птн 06:54:04 #494 №235274 
>>234955
А если так? Даже субкоричневый не закроет?
Аноним 04/12/15 Птн 06:54:21 #495 №235276 
14492012615990.png
>>235274
Аноним 04/12/15 Птн 07:52:47 #496 №235288 
>>235276
universe sandbox нужен тебе
Аноним 04/12/15 Птн 08:10:22 #497 №235294 
>>235276
Я не очень понял суть второго пика, но то, что я хотел тебе сказать, это то чтобы тень от гг полностью закрывала планету, ей нужно находится на расстоянии прямого гравитационного влияния гг. То есть быть его спутником. Если у тела сформировалась собственная орбита, то это значит, что эта орбита достаточно удалена от массивных соседей по системе, и их воздействие уравновесило эту самую орбиту.
Аноним 04/12/15 Птн 12:40:35 #498 №235342 
14492220357980.png
14492220357981.png
>>234950
>>235276
Не пойдет. У Юпитера полная тень конечно длиннее, чем у Земли, но все равно до орбиты Сатурна не достает. А ближе - орбита либо не будет устойчивой, либо вообще стане спутником гг.
Так что только пикприклейтед.

А то что у тебя - это сорт оф прохождение юпитера по диску солнца, вид с кассини. Причем из-за несовпадения плоскостей орбит и больших периодов обращения это настолько редкое явление, что даже я бы посмотрел.
Аноним 04/12/15 Птн 15:47:29 #499 №235393 
>>235342
я эти ваши пикчи совсем не всосал, но со второго? сета у меня возникает вопрос
система вообщем эквивалентная двойной системе - а орбиты планет почему как у одинарной ?
Аноним 04/12/15 Птн 16:52:35 #500 №235408 
>>235393
Второй/третий компонент двойной/тройной системы может быть достаточно отдален от основной звезды, чтобы позволить сформироваться стабильным планетарным орбитам вокруг первого компонента.
Аноним 04/12/15 Птн 17:47:37 #501 №235430 
14492404574200.gif
А у меня тут вопросец возник. Вглядись в эту гифку анон. Видишь там вспышки из левого хвостового блока ОДУ?
Так вот, мне кажется, или он использовал ОДУ даже при посадке? Но зачем?
Аноним 04/12/15 Птн 18:16:48 #502 №235443 DELETED
>>235430
мехджеб забыли выключить
Аноним 04/12/15 Птн 18:59:39 #503 №235466 
>>235430
если есть чем управлять кораблём, пусть и в атмосфере - почему-бы не управлять?
Аноним 04/12/15 Птн 19:12:25 #504 №235473 
>>235466
>>235430
Вопрос не в "зачем", а "почему она у него оставалась". Я если честно был под впечатлением что Буран безмоментно сбрасывал горючку перед реентри, как Шаттл. Чем тяжелей, тем горячей вход ведь, да и ЦТ нужный соблюдать надо, чтобы плоскости справились.
Звездолёты Аноним 04/12/15 Птн 19:50:22 #505 №235488 
14492478227870.jpg
Поясните про фотонный двигатель. Ведь у фотонов нет массы покоя, означает ли это что после разгона общая масса корабля уменьшится?
Если да, то, предположим, использовался химический аккумулятор, куда исчезла масса?
А если нет, то нет ли тут нарушения закона сохранения импульса?
Аноним 04/12/15 Птн 19:51:06 #506 №235489 
>>235488
>>общая масса корабля *не уменьшится
самофикс
Аноним 04/12/15 Птн 20:01:56 #507 №235492 
14492485163810.png
>>235488
Сдается мне, этот человек хочет законы сохранения шатать
Аноним 04/12/15 Птн 21:48:27 #508 №235529 
>>235488
тудаже куда она и раньше исчезала, в искривление просртанства времени
Аноним 05/12/15 Суб 01:10:04 #509 №235580 
Какой гравитационный радиус вселенной?
Аноним 05/12/15 Суб 14:22:31 #510 №235661 
>>235529
Получается масса корабля уменьшится, т.е. заряженный аккумулятор или конденсатор весит чуть больше разряженного?
Аноним 05/12/15 Суб 15:01:37 #511 №235665 
>>235661
да масса акума при разряжении уменьшится.
Аноним 05/12/15 Суб 17:31:09 #512 №235701 
Расскажите про время? Откуда оно взялось после БВ? Почему рядом с черной дырой время замедляется? Почему можно путешествовать во времени вперед, перемещаясь с огромной скоростью?
И кстати, физики ну почти стопроц уверены на счет существования бозона Хиггса, который грубо говоря отвечает за массу. Может быть ли что-нибудь отвечать за время?
Аноним 05/12/15 Суб 20:54:22 #513 №235843 
14493380622970.jpg
14493380622991.jpg
Картинки показывают сколько понадобилось бы времени чтобы долететь со скоростью света до объекта. Вопрос: Почему на последнем пике аж 44 МИЛЛИАРДА лет? Ведь вселенная появилась 13 млрд лет назад, почему чтобы долететь до её конца нужно потратить гораздо больше?
Аноним 06/12/15 Вск 00:23:40 #514 №235889 
>>235843
Потому что она расширяется
Аноним 06/12/15 Вск 00:26:54 #515 №235892 
>>227164 (OP)
А эффект пращи Земной гравитации используется ИРЛ для запусков, например, на Марс там или Юпитер всяких научных телескопов?
Просто в НФ его сейчас довольно часто показывают "Марсианин" для полета на Марс, "Пробуждение Левиафана" там вообще по поясу астероидов и дальним рубежам так перемещаются от спутника к спутнику
Аноним 06/12/15 Вск 11:11:16 #516 №236018 
>>227164 (OP)
Задаю оче интересный ответ: есть две плоскости, и одна из них - плоскость нашей галактики.
Вторая задается тремя точками, центрами галактик, нашей, в треугольнике и в андромеде.

Внимание, правильный вопрос: каков двугранный угол между этими плоскостями?
Аноним 06/12/15 Вск 11:36:17 #517 №236019 
14493909774440.png
Почему ракеты на всякие луны/марсы/точки лагранжа не запускают вертикально вверх, а обязательно выводят сначала на низкую орбиту. В чём профит?
Разве так не тратится лишняя скорость?
Аноним 06/12/15 Вск 12:48:18 #518 №236031 
>>227164 (OP)
Какой материал самый годный для экранирования по сочетанию веса и толщины слоя половинного ослабления, минимальному наличию наведенной радиоактивности?
Аноним 06/12/15 Вск 19:11:20 #519 №236126 
Какое самое большое число, в системе СИ было зарегистрировано во вселенной?
Аноним 06/12/15 Вск 19:11:52 #520 №236127 
>>236031
Вода.
Аноним 06/12/15 Вск 19:41:01 #521 №236139 
>>235137
Спасибо, не читал тред, и даже не предполагал что будет аналогичный вопрос.
Аноним 07/12/15 Пнд 09:55:36 #522 №236347 
Что будет с подводной лодкой и её экипажем на околоземной орбите в течении полугода?
Аноним 08/12/15 Втр 16:22:09 #523 №236735 
14495809294540.png
У меня тут логическая цепочка назрела. Я как то летом ебал тут мозг анонам своей теорией. теперь она получила развитие. Так вот. Смотрите. Свет и сигнал идёт от нашей планеты, например, до другого края, допустим, 1000 лет. То есть через 200 лет на той стороне, куда идёт свет, для этой точки тут тех людей и всего прочего уже в живых не будет. ТАк вот. Это одно. А по второму, получается, не возможно заглянуть в будущее, так как ты бы его смог изменить, и получается ты бы не смог увидеть то, чего не могло бы быть. Я вот картинку нарисовал, чтоб примерно поняли мою мысль. Типо я пришёл к парадоксу. То есть теоретически увидеть будущее можно, но это противоречит моим логическим выводам. То есть тот чел на другой точке будет видеть еще до приближения всё, что Уже произошло и что произойдёт в дальнейшем в течении 1000 лет. А этот чел не сможет увидеть будущее, так как это не возможно. Как вам такой парадокс.
Аноним 08/12/15 Втр 19:53:46 #524 №236882 
14495936261570.jpg
>>236735
Аноним 08/12/15 Втр 20:35:11 #525 №236930 
>>236735
Сукаааааа какой же ты тупой.
>А по второму, получается, не возможно заглянуть в будущее, так как ты бы его смог изменить, и получается ты бы не смог увидеть то, чего не могло бы быть.
Что ты несешь вообще.
Аноним 08/12/15 Втр 21:42:18 #526 №236958 
От земли к марсу, со скоростью 0,7 от скорости света относительно земли, полетел объект.
Почти в то же время от марса к земле полетел аналогичный объект с той же скоростью.
Какова скорость их друг относительно друга?
Аноним 09/12/15 Срд 08:01:58 #527 №237121 
14496373184100.png
>>236958
Посчитай, если не лень.
Аноним 09/12/15 Срд 09:15:59 #528 №237127 
>>236930
Требую разъяснения, раз ты такой умный. Я же написал, в чём парадокс.
Аноним 12/12/15 Суб 01:30:55 #529 №237780 
>>235892
Бамп, что ли.
Аноним 12/12/15 Суб 06:24:42 #530 №237809 
>>235892>>237780
Естественно. Например, вспомни про международный проект "Морской старт" и где Франция разместила свой космодром Куру. Чем ближе запуск к экватору, тем сильнее эффект пращи, тем больше забрасываемый груз и дешевле запуск.

Другой вопрос - используется ли это где-то помимо собственно запусков. На это я не отвечу - просто не представляю, с какими телами в пределах СС Земные КА могут взаимодействовать.
Аноним 12/12/15 Суб 10:46:32 #531 №237826 
>>237780
Используют, но редко. С гравиманевром именно у Земли летел к Меркурию Мессенджер, с ним же планируют отправить Titan Saturn System Mission, если эта штука когда-нибудь взлетит.
Аноним 12/12/15 Суб 17:21:32 #532 №237891 
>>237809
>>237826
Спасибо большое.
Двойные звезды эншт 13/12/15 Вск 12:56:54 #533 №238057 
Помогите написать доклад по этой теме .Еще со школы ненавижу писать сочинения не знаю как начать...(я вообще по адресу)
comments powered by Disqus

Отзывы и предложения